[obm-l] Re: [obm-l] Re: [obm-l] O caráter não enumerável de R

2002-09-10 Por tôpico Nicolau C. Saldanha

On Mon, Sep 09, 2002 at 10:31:07PM +, Laurito Alves wrote:
> O que você chama de N*N*N ???
> 
> Se for um produto cartesiano de N uma quantidade enumerável de vezes, ele é 
> enumerável.

O limite direto das inclusões (usando o sinal < no lugar de está contido)

N < N^2 < N^3 < ...

é o conjunto das seqüências de naturais que são zero a partir de certo
ponto; este conjunto é enumerável. Este conjunto às vezes é chamado
de N^(N) ou N^infty. Mas há outra interpretação: o conjunto N^N
de todas as seqüências de naturais (com qualquer comportamento no infinito)
é não enumerável, tem o cardinal de R.  Na verdade, via frações contínuas,
é fácil construir uma bijeção natural entre este conjunto e o dos números
irracionais.

[]s, N.
=
Instruções para entrar na lista, sair da lista e usar a lista em
http://www.mat.puc-rio.br/~nicolau/olimp/obm-l.html
O administrador desta lista é <[EMAIL PROTECTED]>
=



Re: [obm-l] violencia

2002-09-10 Por tôpico Nicolau C. Saldanha

On Sat, Sep 07, 2002 at 11:45:37PM +, Fernanda Medeiros wrote:
> 
> 
> Olá,
> alguém pode dar uma ajuda nestas questões?
> 1.a)uma "gang" tem infinitos bandidos e cada um dos meliantes tem um único 
> inimigo no interior da "gang",que ele quer matar.Prove q é possivel reunir 
> uma quantidade infinita de bandidos desta "gang", semq  haja  o risco de q 
> um bandido mate outro durante a reunião.
> b)Se cada bandido tiver um nº finito mas indefinido de inimigos(um bandido 
> pode ter 2 inimigos, outro somente 1, um terceiro pode ter 20 e assim por 
> diante).Será sempre possivel promover uma reunião com infinitos bandidos sem 
> risco de derramamento de sangue?

Sei que um monte de gente já respondeu mas gostei da questão e quero
dar a minha solução.

Antes de mais nada devemos entender que no enunciado estão implicitamente
excluídos os bandidos suicidas (inimigos deles próprios)...

(a) Se existe algum bandido x que é odiado por uma infinidade de outros
bandidos, escolhemos todos os bandidos que odeiam x. Supomos a partir
de agora que qualquer bandido é odiado apenas por um número finito
de outros bandidos: assim a presença de um bandido na reunião só exclui
um número finito de outros (o que ele odeia e os que o odeiam).
É bem fácil montar um conjunto infinito: pegue um bandido qualquer,
um que não tenha sido excluido pelo primeiro, um que não tenha sido
excluido pelos dois primeiros,... A qualquer momento apenas um número
finito de bandidos foi excluido logo podemos continuar.

(b) Se os bandidos se chamam 0, 1, 2, 3, ..., n, ... e cada bandido
odeia todos os que têm um número menor do que o dele é impossível
reunir sequer dois bandidos.

Mas o item que me parece mais interessante é:

(c) se cada bandido odeia k outros bandidos (k fixo),
ainda é possível reunir uma infinidade de bandidos?

A resposta ainda é sim e provamos isso por indução em k.
Imagine que cada bandido ordena seus inimigos de 1 a k.
Por indução, podemos formar um subconjunto em que ninguém
odeia ninguém nas (k-1) primeiras posições (mas talvez
odeie na posição k). Acabamos de reduzir o problema ao item (a)...

[]s, N.
=
Instruções para entrar na lista, sair da lista e usar a lista em
http://www.mat.puc-rio.br/~nicolau/olimp/obm-l.html
O administrador desta lista é <[EMAIL PROTECTED]>
=



[obm-l] Re: [obm-l] Re: [obm-l] O caráter não enumerável de R

2002-09-10 Por tôpico Nicolau C. Saldanha

On Tue, Sep 10, 2002 at 04:51:53PM +, Laurito Alves wrote:
> Colegas,
> 
> Ou estou viajando muito ou é certo que a união enumerável de conjuntos 
> enumeráveis é enumerável ?

Sim, a união enumerável de conjunto enumeráveis é enumerável.
> 
> Se me lembro bem, provamos este fato com um argumento semelhante ao que 
> Cantor usou para provar a enumerabilidade de Q.
> 
> Onde está meu erro ?

Você deveria repetir o raciocínio, assim eu não entendo a pergunta...
[]s, N.
=
Instruções para entrar na lista, sair da lista e usar a lista em
http://www.mat.puc-rio.br/~nicolau/olimp/obm-l.html
O administrador desta lista é <[EMAIL PROTECTED]>
=



Re: [obm-l] Continuo Perguntando...

2002-09-10 Por tôpico Nicolau C. Saldanha

On Tue, Sep 10, 2002 at 02:57:34PM -0300, e isso mesmo wrote:
> Amigos, alguém poderia me explicar detalhadamente o que são números
> randômicos e como se constrói esses números? Li isso em algum lugar sobre
> loterias.  Obrigado

Se você está interessado em saber como fazer um algoritmo para gerar
número pseudo-aleatórios, veja o clássico 'The Art of Computer Programming',
de D. Knuth. O assunto é não trivial e muitos softwares que supostamente
geram números aleatórios fazem um péssimo serviço. Pelo que eu sei loterias
não usam computadores, usam aquelas gaiolonas esféricas cheias de bolinhas
que você vê na televisão.

[]s, N.

=
Instruções para entrar na lista, sair da lista e usar a lista em
http://www.mat.puc-rio.br/~nicolau/olimp/obm-l.html
O administrador desta lista é <[EMAIL PROTECTED]>
=



Re: [obm-l] violencia

2002-09-10 Por tôpico Nicolau C. Saldanha

On Tue, Sep 10, 2002 at 09:04:16AM -0300, Nicolau C. Saldanha wrote:
> On Sat, Sep 07, 2002 at 11:45:37PM +, Fernanda Medeiros wrote:
> > 
> > 
> > Olá,
> > alguém pode dar uma ajuda nestas questões?
> > 1.a)uma "gang" tem infinitos bandidos e cada um dos meliantes tem um único 
> > inimigo no interior da "gang",que ele quer matar.Prove q é possivel reunir 
> > uma quantidade infinita de bandidos desta "gang", semq  haja  o risco de q 
> > um bandido mate outro durante a reunião.
> 
> (a) Se existe algum bandido x que é odiado por uma infinidade de outros
> bandidos, escolhemos todos os bandidos que odeiam x. Supomos a partir
> de agora que qualquer bandido é odiado apenas por um número finito
> de outros bandidos: assim a presença de um bandido na reunião só exclui
> um número finito de outros (o que ele odeia e os que o odeiam).
> É bem fácil montar um conjunto infinito: pegue um bandido qualquer,
> um que não tenha sido excluido pelo primeiro, um que não tenha sido
> excluido pelos dois primeiros,... A qualquer momento apenas um número
> finito de bandidos foi excluido logo podemos continuar.

Alguém andou perguntando se esta demonstração precisa do axioma da escolha.
Precisa. Mesmo no caso particular em que os bandidos se odeiam aos pares
(se x odeia y então y odeia x) e que basta pegar um bandido de cada par
estamos usando o axioma da escolha (*qual* bandido de cada par
você vai pegar?).  Isto não deveria incomodar ninguém.
O axioma da escolha é usado implicitamente o tempo todo.

[]s, N.



=========
Instruções para entrar na lista, sair da lista e usar a lista em
http://www.mat.puc-rio.br/~nicolau/olimp/obm-l.html
O administrador desta lista é <[EMAIL PROTECTED]>
=



Re: [obm-l] IRRACIONALIDADE DE PI

2002-09-11 Por tôpico Nicolau C. Saldanha

On Wed, Sep 11, 2002 at 02:40:20PM -0300, Jorge Paulino wrote:
> Galera, alguém conhece alguma demonstração,
> acessível a alunos de segundo grau, da irracionalidade
> do número PI?
> Um Abraço,
> Jorge

Não acho que *exista* uma demonstração da irracionalidade
de pi que seja acessível a alunos (típicos) de segundo grau.
Mas há uma demonstração curta usando cálculo em 'Proofs
from the book' de Aigner e Ziegler (há uma tradução bem recente).
Outra referência é 'Irrational numbers' de Ivan Niven.

[]s, N.
=
Instruções para entrar na lista, sair da lista e usar a lista em
http://www.mat.puc-rio.br/~nicolau/olimp/obm-l.html
O administrador desta lista é <[EMAIL PROTECTED]>
=



Re: [obm-l] TESTE

2002-09-11 Por tôpico Nicolau C. Saldanha

On Wed, Sep 11, 2002 at 03:01:50PM -0300, Hely Jr. wrote:
> Observem a sequência abaixo:
> 
> 2 - 10 - 12 - 16 - 17 - 18 - 19 ..
> 
> E agora, respondam: qual o próximo número da sequência???
> 
> a) 20
> b) 22
> c) 175
> d) 200
> 
> Existe alguma pegadinha neste exercício?

Existe. Este problema é fácil para turmas de alfabetização (no Brasil!)
mas difícil para matemáticos... ;-)

[]s, N.

=
Instruções para entrar na lista, sair da lista e usar a lista em
http://www.mat.puc-rio.br/~nicolau/olimp/obm-l.html
O administrador desta lista é <[EMAIL PROTECTED]>
=



Re: [obm-l] Axioma da Escolha

2002-09-12 Por tôpico Nicolau C. Saldanha

On Wed, Sep 11, 2002 at 04:01:42PM -0300, 498 - Artur Costa Steiner wrote:
> Nos últimos dias o Axioma da Escolha foi bastante mencionado nesta 
> lista, motivado por um interessante problema (violência), sugerido por 
> uma das participantes, e que involve este axioma.
> 
> Eu não estou certo, mas, no meio matemático, ainda existem hoje 
> restrições a este axioma, no sentido de que alguma prova nele baseada 
> possa ser considerada questionável ou mesmo inválida?

Este assunto mereceria uma resposta mais longa, mas o axioma da escolha
é 'aceito' no sentido seguinte: a maioria dos matemáticos usa este
axioma sem parar para pensar no assunto. Aliás sem nem saber direito
quando está realmente usando o axioma. Alguns matemáticos, entretanto,
especialmente especialistas em lógica ou teoria dos conjuntos, acham
interessante notar exatamente quando o tal axioma é utilizado.

[]s, N.
=
Instruções para entrar na lista, sair da lista e usar a lista em
http://www.mat.puc-rio.br/~nicolau/olimp/obm-l.html
O administrador desta lista é <[EMAIL PROTECTED]>
=



Re: [obm-l] Problema de um amigo

2002-09-17 Por tôpico Nicolau C. Saldanha

On Mon, Sep 16, 2002 at 11:48:06PM -0300, Edmilson wrote:
> Dividir uma circunferência em 9 partes iguais, porprocesso geométrico

Se por 'processo geométrico' você quer dizer usando apenas régua e compasso
então é impossível. É um resultado clássico de álgebra que o círculo pode
ser dividido em n partes iguais com régua e compasso apenas se n é da forma

n = 2^k * p_1 * p_2 * ... * p_m

onde p_1 < p_2 < ... < p_m são primos da forma p = 1 + 2^(2^a).
Os únicos primos conhecidos desta forma são 3, 5, 17, 257 e 65537;
existem uns argumentos que tornam plausível que estes sejam *todos*.

Quando eu estudei desenho geométrico no ginásio apredíamos métodos
para dividir o círculo em 5, 6, 7 ou 9 partes iguais mas era na decoreba
sem explicação nenhuma e sem que ninguém mencionasse que uns desses
processos (5 e 6) eram exatos e outros (7 e 9) não.

[]s, N.
=
Instruções para entrar na lista, sair da lista e usar a lista em
http://www.mat.puc-rio.br/~nicolau/olimp/obm-l.html
O administrador desta lista é <[EMAIL PROTECTED]>
=



Re: [obm-l] Conjuntos Fechados

2002-09-19 Por tôpico Nicolau C. Saldanha

On Tue, Sep 17, 2002 at 01:43:59PM -0300, Humberto Naves wrote:
> Oi,
> 
>   Li num livro de análise, que o conjunto dos irracionais não pode ser escrito
> como uma união enumerável de fechados. Como demostrar esse fato?

Isto é essencialmente o teorema de Baire.
Uma união enumerável de fechados de interior vazio ainda tem interior
vazio. Para provar isso vá tomando intervalos compactos encaixados
disjuntos de cada fechado da seqüência.

[]s, N.
=
Instruções para entrar na lista, sair da lista e usar a lista em
http://www.mat.puc-rio.br/~nicolau/olimp/obm-l.html
O administrador desta lista é <[EMAIL PROTECTED]>
=



Re: [obm-l] combinatoria

2002-09-21 Por tôpico Nicolau C. Saldanha

On Sat, Sep 21, 2002 at 02:28:22PM -0300, adr.scr.m wrote:
> estou com uma duvida nessa questao,porque eu 
> fiz usando o 1ºlema de Kaplansky,e meu 
> professor disse que nao pode:
> (IME-75/76) Considere uma turma com n 
> alunos,numerados de 1 a n.Deseja-se 
> organizar uma comissao de 3 alunos.De 
> quantas maneiras pode ser formada esta 
> comissao,de modo que nao facam parte da 
> mesma 2 ou 3 alunos designados por numeros 
> consecutivos ?
> []'s.
> Adriano.

Eu contaria todas as comissões, o que dá binomial(n,3) = n(n-1)(n-2)/6,
e depois excluiria as da forma j,k,k+1. Para contá-las, eu primeiro
escolho k (temos (n-1) valores possíveis) e depois j ((n-2) valores
possíveis), o que dá (n-1)(n-2); note entretanto que as n-2 comissões
da forma i,i+1,i+2 foram eliminadas duas vezes (j=i,k=i+1 e j=i+2,k=i)
e isso deve ser corrigido.

Assim, temos

n(n-1)(n-2)/6 - (n-1)(n-2) + (n-2) = (n-2)(n(n-1) - 6(n-2))/6

o que dá os valores certos para n=2,3,4 (0), n=5 (1) e n=6 (4).
Como a resposta obviamente é um polinômio de grau <= 3 estes 5 testes
são bem convincentes. :-)

[]s, N.
=
Instruções para entrar na lista, sair da lista e usar a lista em
http://www.mat.puc-rio.br/~nicolau/olimp/obm-l.html
O administrador desta lista é <[EMAIL PROTECTED]>
=



Re: [obm-l] Probabilidade

2002-09-22 Por tôpico Nicolau C. Saldanha

On Sat, Sep 21, 2002 at 07:43:50PM -0300, Wagner wrote:
> Oi para todos !

>   É possível descrever a probabilidade do evento abaixo em uma fórmula apenas?
> 
> Uma caneta é girada por uma pessoa de forma aleatória. Os movimentos possíveis 
>da caneta são meia volta para a esquerda e meia volta para a direita. Qual a 
>probabilidade de que após n movimentos aleatórios a caneta tenha feito pelo menos 2 
>voltas ou para esquerda ou para a direita (considere apenas o balanço final, ex: 2 
>voltas para esquerda e meia volta para direita deve ser considerado como uma volta e 
>meia para a esquerda).
> 
> André T.

Devemos separar os casos par e ímpar, e é mais fácil calcular a probabilidade
de que a caneta tenha girado *menos* de duas voltas e depois fazer 1-...

Se n = 2m, a probabilidade de que a caneta tenha dado menos de 2 voltas
é a probabilidade de que ela tenha feito m-1, m ou m+1 movimentos para a
esquerda, o que tem probabilidade

(binomial(2m,m-1) + binomial(2m,m) + binomial(2m,m+1))/2^(2m)

Analogamente, se n = 2m+1, a probabilidade é

(binomial(2m+1,m-1) + binomial(2m+1,m) +
 binomial(2m+1,m+1) + binomial(2m+1,m+2))/2^(2m+1)

[]s, N.
=
Instruções para entrar na lista, sair da lista e usar a lista em
http://www.mat.puc-rio.br/~nicolau/olimp/obm-l.html
O administrador desta lista é <[EMAIL PROTECTED]>
=



[obm-l] Re: [obm-l] O número gugol

2002-09-23 Por tôpico Nicolau C. Saldanha

On Sun, Sep 22, 2002 at 07:07:22PM -0300, Wagner wrote:
> Oi pessoal!
> 
> Li em uma reportagem que um tal de número gugol é 10^100 e que outro número
> chamado de gugolplex é igual à gugol^gugol. Fiquei pensando, o que seria
> maior, (1gugol)! ou 1 gugolplex. Como acho a resposta disso?

Acho que o usual é definir um gugolplex como 10^(10^100), mas o fato é que

10^(10^100) < (10^100)! < (10^100)^(10^100)

pois (10^100)! ~ (10^100)^(10^100) e^(- 10^100) sqrt(2 pi 10^100)

o que é claramente menor que (10^100)^(10^100) pois
e^(- 10^100) sqrt(2 pi 10^100) é bem pequeno.

Por outro lado,

(10^100)! ~ 10^(10^100) (10^99)^(10^100) e^(- 10^100) sqrt(2 pi 10^100)

e como 10^99 é muito maior do que e,
(10^99)^(10^100) e^(- 10^100) sqrt(2 pi 10^100) é muito maior do que 1.

[]s, N.
=
Instruções para entrar na lista, sair da lista e usar a lista em
http://www.mat.puc-rio.br/~nicolau/olimp/obm-l.html
O administrador desta lista é <[EMAIL PROTECTED]>
=



Re: [obm-l] determinantes

2002-09-23 Por tôpico Nicolau C. Saldanha

On Sun, Sep 22, 2002 at 10:33:45AM -0300, haroldo wrote:
> Gostaria de ajuda no cálculo desse determinante:
>  
>  
>  1 1/21/3      1/n
> 1/2   1/31/4  1/(n+1)
> 1/3   1/41/5...   1/(n+2)
>  ..
> ..
> 1/n   1/(n+1)  ... .  1/(2n-1)

Esta é a chamada matriz de Hilbert.
Você encontrará mais informações sobre ela
em bons livros de álgebra linear ou em
http://mathworld.wolfram.com/HilbertMatrix.html

Se chamarmos o determinante da matriz de Hilbert nxn de 1/f(n) temos

f(n)/f(n-1) = (2n-1) (binomial(2n-2,n-1))^2 

o que nos dá uma fórmula um pouco complicada para os determinantes...

A seqüência f(n) é formada pelos números inteiros positivos abaixo:

1
12
2160
6048000
26671680
18631342033920
206790904792577064960
36535684712573448587811225600
10287817843785696978870529629093888
462068939479146913162956288390362787269836800
331225048970634137553621436270407271060801276724694220800
3791065794363045171518854790347963918801886878641184641043243047321600

e esta é a seqüência A005249 na excelente Encyclopaedia of Integer Sequences:

http://www.research.att.com/~njas/sequences/

[]s, N.
=
Instruções para entrar na lista, sair da lista e usar a lista em
http://www.mat.puc-rio.br/~nicolau/olimp/obm-l.html
O administrador desta lista é <[EMAIL PROTECTED]>
=



Re: [obm-l] EDO - nenhum palpite?

2002-09-24 Por tôpico Nicolau C. Saldanha

On Tue, Sep 24, 2002 at 09:18:48AM -0300, bruno lima wrote:
> Numa dessas provas universitarias por ai apareceu:
> 
> y''=y*exp(x) 
> 
> y: Vai de [a,b]em R , e y(a)=0 e y(b)=0.
> 
> Estou errado ou a unica solucáo é a identicamente
> nula??

Sim, a única solução é identicamente nula.
Podemos ver isso resolvendo y(a) = 0, y'(a) = 1.
É fácil ver que y(t) > t-a para todo t > a e portanto y(b) > 0.

Você tem certeza que a equação é esta? A coisa seria bem diferente
se fosse, por exemplo,

y'' = - y * exp(x)

ou até

y'' = exp(y)

[]s, N.
=
Instruções para entrar na lista, sair da lista e usar a lista em
http://www.mat.puc-rio.br/~nicolau/olimp/obm-l.html
O administrador desta lista é <[EMAIL PROTECTED]>
=



Re: [obm-l] Racionais diadicos

2002-09-25 Por tôpico Nicolau C. Saldanha

On Wed, Sep 25, 2002 at 02:52:23PM -0300, Johann Peter Gustav Lejeune Dirichlet wrote:
> O que sao racionais diadicos?

Um racional cujo denominador é uma potência de 2. []s, N.
=
Instruções para entrar na lista, sair da lista e usar a lista em
http://www.mat.puc-rio.br/~nicolau/olimp/obm-l.html
O administrador desta lista é <[EMAIL PROTECTED]>
=



Re: [obm-l] Jogos

2002-09-27 Por tôpico Nicolau C. Saldanha

On Thu, Sep 26, 2002 at 11:34:46PM -0300, Luiz Felipe Constantino wrote:
> outro dia, vi em algum lugar o responsável pela tabela dos jogos do 
> Campeonato Brasileiro dizer que com os 26 times não era possível fazer com 
> que todos os times jogassem na mesma rodada durante todo o campeonato. Ou 
> seja, não dava para ter treze jogos em toda rodada, de modo que cada time 
> jogue com o outro apenas uma vez. Gostaria de saber como colocar isso no 
> papel (como provar)...
> 
> L. Felipe

Acho que este senhor usou a palavra 'impossível' no sentido não matemático,
querendo dizer que não há campos de futebol de boa qualidade um número
suficiente, ou que os jogadores estão ocupados treinando pela seleção,
ou alguma outra consideração não matemática do gênero...

[]s, N.
=
Instruções para entrar na lista, sair da lista e usar a lista em
http://www.mat.puc-rio.br/~nicolau/olimp/obm-l.html
O administrador desta lista é <[EMAIL PROTECTED]>
=



Re: [obm-l] Corpos tetradimensionais

2002-09-27 Por tôpico Nicolau C. Saldanha

On Thu, Sep 26, 2002 at 08:47:50PM -0300, Wagner wrote:
> Oi para todos!
> 
> Queria saber se um corpo tridimensional pode ser definido em um plano de 4
> dimensões como sendo a intersecção de 2 corpos tetradimensionais. Ex: O
> sistema : x^2+y^2+z^2+w^2=16 ( I ) e x.y.z-w =3

Não, em geral duas equações em R^4 definem um objeto de dimensão 2 e não 3.
Note que *uma* equação em 4 variáveis já define um objeto de dimensão 3.
Por outro lado, duas *inequações* em R^4 definem um objeto de dimensão 4.

[]s, N.
=
Instruções para entrar na lista, sair da lista e usar a lista em
http://www.mat.puc-rio.br/~nicolau/olimp/obm-l.html
O administrador desta lista é <[EMAIL PROTECTED]>
=



Re: [obm-l]

2002-09-27 Por tôpico Nicolau C. Saldanha

On Fri, Sep 27, 2002 at 12:19:05AM -0300, Mário Pereira wrote:
> Preciso de ajuda:
> 
> A escala termométrica Celsius adota os valores de 0 a 100 para pontos de
> fusão do gelo e de ebulição da água à pressão normal, respectivamente. A
> escala Fahrenheit adota os valores 32 e 212 para esses mesmos pontos. Então a
> partir de uma determinada temperatura o nº lido na escala Fahrenheit é maior
> que o lido na escala Celsius. Que temperatura é essa?

-40

Basta interpolar por funções da forma ax+b.

[]s, N.
=
Instruções para entrar na lista, sair da lista e usar a lista em
http://www.mat.puc-rio.br/~nicolau/olimp/obm-l.html
O administrador desta lista é <[EMAIL PROTECTED]>
=



Re: [obm-l] Corpos tetradimensionais

2002-09-27 Por tôpico Nicolau C. Saldanha

On Thu, Sep 26, 2002 at 09:37:53PM -0300, Artur Costa Steiner wrote:
>  
> Oi para todos!
>  
> Queria saber se um corpo tridimensional pode ser definido em um plano de
> 4 dimensões como sendo a intersecção de 2 corpos tetradimensionais. Ex:
> O sistema : x^2+y^2+z^2+w^2=16 ( I ) e x.y.z-w =3
>  
> André T.
>  
> Na realidade, não existem corpos de dimensão superior a 2, entendendo-se
> como corpo a estrutura algébrica que satisfaz àqueles conhecidos axiomas
> referentes à soma e à multiplicação.

Acho que o André *não* estava pensando na definição algébrica de corpo...

[]s, N.
=
Instruções para entrar na lista, sair da lista e usar a lista em
http://www.mat.puc-rio.br/~nicolau/olimp/obm-l.html
O administrador desta lista é <[EMAIL PROTECTED]>
=



Re: [obm-l] Corpos tetradimensionais

2002-09-29 Por tôpico Nicolau C. Saldanha

On Fri, Sep 27, 2002 at 02:28:32PM -0300, 498 - Artur Costa Steiner wrote:
> Já que falamos em corpos, gostaria de tirar a seguinte dúvida. Li no 
> newsgroup sci.math que, embora não existam corpos de dimensão finita 
> >2, há corpos de dimensãoinfinita.  Isto é verdade? Se for, como 
> podemos definir um corpo infinito-dimensional? Sei que existem espaços 
> vetoriais de dimensão infinita, mas não sabia que existiam corpos assim.

Um exemplo é o das funções racionais com coeficientes reais ou complexos.
Outro exemplo é o das funções meromorfas em algum aberto A contido em C.

Uma função é meromorfa em A se ela for holomorfa em A menos um número
finito de pontos sendo que todos estes pontos são polos.

[]s, N.
=
Instruções para entrar na lista, sair da lista e usar a lista em
http://www.mat.puc-rio.br/~nicolau/olimp/obm-l.html
O administrador desta lista é <[EMAIL PROTECTED]>
=



Re: [obm-l] 0,999.....

2002-09-29 Por tôpico Nicolau C. Saldanha

On Sat, Sep 28, 2002 at 11:45:41AM -0300, Artur Costa Steiner wrote:
> Aqueles que costumam acessar o newsgroup internacional sci.math
> provavvelmente já notaram que a pergunta "É 0,9 = 1" já apareceu
> zilhões de vezes e que, toda vez que é reapresentada, provoca intensa
> polêmica, levando a cadeias de resposta com 100 ou mais mensagens.
> Interessante que esta discussão acaba fatalmente indo para aspectos
> topológicos,  e mesmo filosóficos, ligados à construção dos reais e
> assuntos correlatos.
>  
> Há motivo para tanta polêmica assim? Na realidade, 0,... é apenas
> uma representação gráfica da série geométrica 9/10 + 9/100 +
> ...9/10^n..., cujo limite é 1. 0,... =1 se, conforme freqüentemente
> se faz, interpretarmos 0,999 como o limite da série, e não como a
> série em si, da mesma forma como se diz que 1 + 1/2!... + 1/n!. =e.
> Qual a razão para tanta polêmica a respeito deste assunto.

Já se falou muito sobre este tema aqui, veja uma lista de links em
http://www.mat.puc-rio.br/~nicolau/olimp/obm-l.200208/msg00112.html

Aliás obrigado ao Eduardo Casagrande Stabel por preparar esta lista.

[]s, N.
=
Instruções para entrar na lista, sair da lista e usar a lista em
http://www.mat.puc-rio.br/~nicolau/olimp/obm-l.html
O administrador desta lista é <[EMAIL PROTECTED]>
=



[obm-l] Re: [obm-l] Re: Menor distância à circunferência

2002-09-29 Por tôpico Nicolau C. Saldanha

On Sat, Sep 28, 2002 at 01:03:12PM -0300, tarsis19 wrote:
>  Para saber o ponto de menor distância da circunferência 
> (x-4)^2 + (y-3)^2 = 9 à origem, deve-se achar a reta que 
> passa pela origem e pelos pontos (4,3);(y=3x/4); e fazer 
> a interseção dela com a circunferência. 

Mas se você só quer o valor da distância, basta fazer 5 - 3 = 2,
onde 5 é a distância da origem ao centro (4,3) e 3 é o raio...

[]s, N.
=
Instruções para entrar na lista, sair da lista e usar a lista em
http://www.mat.puc-rio.br/~nicolau/olimp/obm-l.html
O administrador desta lista é <[EMAIL PROTECTED]>
=



[obm-l] Re: [obm-l] Função Analítica

2002-09-29 Por tôpico Nicolau C. Saldanha

On Sat, Sep 28, 2002 at 12:58:45PM -0300, Salvador Addas Zanata wrote:
> 
> A definicao de analiticidade pra funcoes complexas implica no seguinte
> fato:
> 
> Se uma funcao complexa f e analitica num ponto, entao o seu polinomio de
> taylor centrado nesse ponto converge para f numa bola suficientemente
> pequena, centrada nesse ponto. 
> 
> 
> Esse fato se obtem por derivacoes da formula integral de Cauchy...
> 
> Pra funcoes f de R^n em R, por exemplo, diz-se que uma tal e analitica
> (num ponto) se o seu polinomio de Taylor (centrado nesse
> ponto) converge para f (numa vizinhanca do ponto).
> 
> Por exemplo, arctan(x) e analitica em x=0, apesar de que seu polinomio de
> Taylor:
> 
> 
> p(x)=x-x^3/3+x^5/5-x^7/7+x^9/9-
> 
> so converge para |x|<1.
> 
> 
> Por outro lado, f(x)=exp(-1/x^2), se x<>0
> f(0)=0 
> 
> E infinitamente diferenciavel no zero, se definirmos todas as derivadas no
> zero como sendo zero. (apesar das derivadas nao serem continuas no zero, o
> limite de todas f'''''''''(x)->0, para x->0). E claro que essa f nao e
> analitica, porque o seu polinomio de Taylor centrado no zero e
> identicamente nulo e a funcao f so se anula em x=0.

Outra definição equivalente é a seguinte: uma função f: A -> R,
A um subconjunto aberto de R^n é real analítica se existir uma 
função complexa analítica g: B -> C, B um aberto de C^n, A contido em B,
g restrita a A igual a f.

[]s, N.
=====
Instruções para entrar na lista, sair da lista e usar a lista em
http://www.mat.puc-rio.br/~nicolau/olimp/obm-l.html
O administrador desta lista é <[EMAIL PROTECTED]>
=



Re: [obm-l] ???

2002-09-30 Por tôpico Nicolau C. Saldanha

On Sun, Sep 29, 2002 at 02:09:24PM -0300, Eder wrote:
> Olá,
> 
> 
> 
> Gostaria de ajuda na situação abaixo.
> 
> 
> Sendo a,b e c inteiros positivos,resolva: (1+1/a)(1+1/b)(1+1/c)=2.
> 
> Eu estava tentando e não me pareceu haver solução...

Podemos supor 1 < a <= b <= c.
Devemos ter a <= 3 pois se a >= 4 temos b, c >= 4 e
(1+1/a)(1+1/b)(1+1/c) <= (1+1/4)(1+1/4)(1+1/4) = 125/64 < 128/64 = 2.
Vamos estudar separadamente os casos a = 2 e a = 3.

Para a = 2 temos 2 <= b <= c e
(1+1/b)(1+1/c) = 4/3
donde devemos ter b <= 6 pois se b >= 7 temos c >= 7 e
(1+1/b)(1+1/c) <= (1+1/7)(1+1/7) = 64/49 < 64/48 = 4/3.
Assim basta testar b = 2, 3, 4, 5, 6 e ver para quais casos
temos c inteiro positivo. Os casos b=2 e b=3 falham mas os outros
nos dão as soluções:

2,4,15
2,5,9
2,6,7

Para a = 3 temos 3 <= b <= c e
(1+1/b)(1+1/c) = 3/2
donde devemos ter b <= 4 pois se b >= 5 temos c >= 5 e
(1 + 1/b)(1+1/c) <= (1+1/5)(1+1/5) = 36/25 < 36/24 = 3/2.
Novamente basta testar b = 3 e b = 4 para obter as soluções

3,3,8
3,4,5

e estas são (a menos de permutações) todas as soluções.

O problema fica mais interessante se permitirmos mais variáveis:

(1+1/x1)...(1+1/xn) = 2, 1 < x1 <= ... <= xn inteiros.
As soluções com n= 4 são

2,4,16,255
2,4,17,135
2,4,18,95
2,4,19,75
2,4,20,63
2,4,21,55
2,4,23,45
2,4,25,39
2,4,27,35
2,4,30,31
2,5,10,99
2,5,11,54
2,5,12,39
2,5,14,27
2,5,15,24
2,5,18,19
2,6,8,63
2,6,9,35
2,6,11,21
2,6,14,15
2,7,7,48
2,7,8,27
2,7,9,20
2,7,12,13
2,8,9,15
2,9,10,11
3,3,9,80
3,3,10,44
3,3,11,32
3,3,12,26
3,3,14,20
3,3,16,17
3,4,6,35
3,4,7,20
3,4,8,15
3,4,10,11
3,5,5,24
3,5,6,14
3,5,8,9
3,6,7,8
4,4,5,15
4,5,5,9
4,5,6,7

Esta lista foi gerada com maple, eu não sabia que ia ficar tão grande...

[]s, N.
=====
Instruções para entrar na lista, sair da lista e usar a lista em
http://www.mat.puc-rio.br/~nicolau/olimp/obm-l.html
O administrador desta lista é <[EMAIL PROTECTED]>
=



[obm-l] Re: [obm-l] Re: [obm-l] qual o próximo número?

2002-10-01 Por tôpico Nicolau C. Saldanha

On Tue, Oct 01, 2002 at 08:10:33PM -0300, Eduardo Casagrande Stabel wrote:
> From: "Juliana Löff" <[EMAIL PROTECTED]>
> >Qual o próximo número?
> >2, 12, 1112, 3112, ?

132112, 1113122112, 311311222112, 13211321322112, 1113122113121113222112, ...

(espero não ter errado)

Leia em voz alta uma seqüência de algarismos para obter a próxima:

3112 é um 3, dois 1 e um 2 (132112)

Conway já estudou esta seqüência.

Como sempre, minha recomendação é dar uma olhada na excelente
enciclopédia de seqüências de inteiros:

http://www.research.att.com/~njas/sequences/

[]s, N.
=
Instruções para entrar na lista, sair da lista e usar a lista em
http://www.mat.puc-rio.br/~nicolau/olimp/obm-l.html
O administrador desta lista é <[EMAIL PROTECTED]>
=



[obm-l] Re: your mail

2002-10-02 Por tôpico Nicolau C. Saldanha

On Wed, Oct 02, 2002 at 10:14:17AM -0300, Guilherme Rubert Pereira wrote:
> Olá amigos da lista:
> 
> Gostaria de uma ajuda na solução da seguinte questão:
> 
> Um pai tem dois filhos, de 2 e 4 anos. Ele prometeu dividir sua fazenda 
> entre os filhos de modo diretamente proporcional às suas idades assim que se 
> case o mais velho dos filhos. Quanto mais tarde este filho se casar, a 
> fração da fazenda que lhe caberá será:
> 
> (a) maior e nunca será menor que 2/3 da fazenda
> (b) maior, mas nunca será maior que 2/3 da fazenda
> (c) menor, mas sempre será maior do que a metade da fazenda
> (d) menor, podendo ser menor do que a metade da fazenda
> (e) igual a 2/3 da fazenda, independente da data do seu casamento.
> 
> Obrigado!
> 
> Guilherme

Se o filho mais velho casa com x+1 anos o filho mais moço terá x-1 anos
e as frações serão

(x+1)/(2x) = 1/2 + 1/2x para o mais velho,
(x-1)/(2x) = 1/2 - 1/2x para o mais jovem.

Assim o correto é a letra:
(c) menor, mas sempre será maior do que a metade da fazenda

[]s, N.


=
Instruções para entrar na lista, sair da lista e usar a lista em
http://www.mat.puc-rio.br/~nicolau/olimp/obm-l.html
O administrador desta lista é <[EMAIL PROTECTED]>
=



Re: [obm-l] CUIDADO COM MENSAGEM

2002-10-02 Por tôpico Nicolau C. Saldanha

Não achei nenhuma mensagem correspondente à descrição e que pudesse
estar infectada nos arquivos. Existe a mensagem
http://www.mat.puc-rio.br/~nicolau/olimp/obm-l.200209/msg00490.html
que é do Paulo Rodrigues, tem subject: Re: [obm-l] Jogos
mas que é claramente inocente, é puro texto.

Algumas mensagens forjam o cabeçário para *fingir* que estão vindo
da lista quando na verdade nunca passaram por aqui, talvez este seja
o caso.

Só para repetir a política da lista quanto a vírus e similares:

* Algumas precauções básicas são tomadas pelo majordomo, o programa
  que administra a lista. Entre elas:

  . só podem ir para a lista mensagens que (aparentemente) venham de
um membro da lista,

  . são proibidas mensagens com attachments grandes.

  Estas precauções de fato fazem vir parar na minha caixa de correio
  várias mensagens suspeitas por dia. A maioria é spam; algumas são
  enviadas por pessoas inocentes mas que não prestaram muita atenção
  às instruções da lista e mandam figuras enormes ou attachments *.doc;
  algumas provavelmente são vírus (eu jogo fora sem tentar descobrir).

* A responsabilidade de defender o seu computador é do usuário.
  Ele pode usar antivírus, clientes de e-mail mais inteligentes,
  sistemas operacionais mais seguros, ler o e-mail num cybercafe,
  usar um computador sem disco rígido ou simplesmente reinstalar tudo
  toda vez que aparecer um vírus. O problema e a opção são dele
  (ou seja, de vocês).

* Esta lista não tem como objetivo discutir segurança de computadores,
  existem outras para isso. Limitem-se a um aviso objetivo (como o do
  Morgado) quando necessário.

* Em nenhum caso nem eu nem a OBM somos responsáveis por qualquer dano
  causado por um vírus ou similar mesmo que tenha passado pela lista.

Obrigado pela atenção, desculpem a chatice, []s, N.

On Wed, Oct 02, 2002 at 08:50:38AM -0300, Augusto César Morgado wrote:
> Aos que nao estao com antivirus atualizado:
> Nao abram o anexo de uma mensagem sobre JOGOS supostamente enviada pelo 
> Paulo Rodrigues (pauloemanu).
> Esta infectado.
> Morgado
> 
> =
> Instruções para entrar na lista, sair da lista e usar a lista em
> http://www.mat.puc-rio.br/~nicolau/olimp/obm-l.html
> O administrador desta lista é <[EMAIL PROTECTED]>
> =
=
Instruções para entrar na lista, sair da lista e usar a lista em
http://www.mat.puc-rio.br/~nicolau/olimp/obm-l.html
O administrador desta lista é <[EMAIL PROTECTED]>
=



Re: [obm-l] POR FAVOR ME AJUDEM EM POSTSCRIPT

2002-10-07 Por tôpico Nicolau C. Saldanha

On Mon, Oct 07, 2002 at 04:18:34PM -0300, Jose Francisco Guimaraes Costa wrote:
> "O dvi se nao me engano e o software do Pato Donald Knuth.E o TeX" 

Arquivos *.dvi são produzidos pelo TeX (ou um de seus descendentes diretos,
como o LaTeX). Donald Knuth é o nome do autor original do TeX.

Uma solução radical para lidar com os formatos *.tex, *.dvi e *.ps
é instalar [Gnu-]Linux (isto pode ser feito em paralelo com alguma versão
de Windows ou MacOS que você já tenha; este processo chama-se dual boot)
pois ele vem (em qualquer boa distribuição) com o tetex, o gv e os outros
programas relevantes. Se você usar Windows e isso parecer complicado demais
aponte seu browser para www.cygwin.com e instale as versão para Windows
dos respectivos programas.

[]s, N.
=
Instruções para entrar na lista, sair da lista e usar a lista em
http://www.mat.puc-rio.br/~nicolau/olimp/obm-l.html
O administrador desta lista é <[EMAIL PROTECTED]>
=



Re: [obm-l] probabilidade

2002-10-07 Por tôpico Nicolau C. Saldanha

On Mon, Oct 07, 2002 at 04:10:55PM -0300, Jose Francisco Guimaraes Costa wrote:
> Eu não havia notado que era uma pegadinha.  

Eu não participei da discussão até agora. Mas devo notar que este problema
não é uma "pegadinha", é um problema clássico que muitos resolvem errado.
A resposta certa é 2/3; a resposta 1/2 é um erro comum.
É comum também que as pessoas que fazem o raciocínio errado se obstinem
e persistam no erro...

Este problema está discutido no meu artigo
"Como perder amigos e enganar pessoas",
publicado na Eureka #1 e disponível na minha home page.
O problema do bode também está lá.

[]s, N.
=
Instruções para entrar na lista, sair da lista e usar a lista em
http://www.mat.puc-rio.br/~nicolau/olimp/obm-l.html
O administrador desta lista é <[EMAIL PROTECTED]>
=



Re: En: [obm-l] probabilidade

2002-10-08 Por tôpico Nicolau C. Saldanha

On Tue, Oct 08, 2002 at 11:35:21AM -0300, Jose Francisco Guimaraes Costa wrote:
> Problemas clássicos não contém armadilhas.

Claro que você está errado, o que torna o problema clássico é exatamente
o fato de pessoas errarem e teimarem no erro.

[]s, N.
=
Instruções para entrar na lista, sair da lista e usar a lista em
http://www.mat.puc-rio.br/~nicolau/olimp/obm-l.html
O administrador desta lista é <[EMAIL PROTECTED]>
=



[obm-l] Re: [obm-l] equação

2002-10-15 Por tôpico Nicolau C. Saldanha

On Tue, Oct 15, 2002 at 05:24:27PM +, Marcelo Souza wrote:
> Olá,
> Dá pra ver que duas soluçoes triviais sao 4 e 2, porem, ha mais uma soluçao 
> naum trivial. Faça o seguinte, desenhe o grafico de x^2 e de 2^x e observe 
> que há mais um ponto de corte alem de 2 e 4...Tente o maple, ou algum 
> programa que construa graficos para uma melhor visualizaçao.
> faliow
> []'s Marcelo

A terceira solução é aproximadamente -0.746960.
Um estudo dos sinais das derivadas demonstra que existem exatamente
estas três raizes reais.
[]s, N.
> 
> >From: "Wagner" <[EMAIL PROTECTED]>
> >Reply-To: [EMAIL PROTECTED]
> >To: <[EMAIL PROTECTED]>
> >Subject: [obm-l] equação
> >Date: Mon, 14 Oct 2002 15:14:02 -0300
> >
> >Oi pessoal!
> >
> >Alguém pode me ajudar com a equação abaixo?
> >2^x=x^2
> >
> >Obrigado
> >
> >André T.
> 
> 
> _
> MSN Photos is the easiest way to share and print your photos: 
> http://photos.msn.com/support/worldwide.aspx
> 
> =====
> Instruções para entrar na lista, sair da lista e usar a lista em
> http://www.mat.puc-rio.br/~nicolau/olimp/obm-l.html
> O administrador desta lista é <[EMAIL PROTECTED]>
> =====
=
Instruções para entrar na lista, sair da lista e usar a lista em
http://www.mat.puc-rio.br/~nicolau/olimp/obm-l.html
O administrador desta lista é <[EMAIL PROTECTED]>
=



Re: [obm-l] terceira fase

2002-10-15 Por tôpico Nicolau C. Saldanha

On Sun, Jan 16, 2000 at 01:24:26PM -0200, gabriel wrote:
> Ola a todos,
> estou com duvida sobre o horario.Esta fase tambem sera realizada
> apartir das 14 horas?

Esta é a instrução da OBM aos coordenadores regionais.
Note que estamos falando de 14 horas no horário de Brasília.

[]s, N.
=
Instruções para entrar na lista, sair da lista e usar a lista em
http://www.mat.puc-rio.br/~nicolau/olimp/obm-l.html
O administrador desta lista é <[EMAIL PROTECTED]>
=



[obm-l] Re: [obm-l] Funções bijetoras(ERRATA)

2002-10-15 Por tôpico Nicolau C. Saldanha

On Tue, Oct 15, 2002 at 01:31:28PM -0300, Wagner wrote:
>Alguém sabe se a afirmação abaixo é verdadeira?
>
>Se uma função é bijetora,
>isso implica que todas as raízes de sua derivada formarem um par
>E se todas as raízes de uma função formarem um par, sua integral é bijetora.
>OBS:Excluindo as funções que possuam raízes além do par.
>
>Considerando como raiz da função, os valores de x tais que f(x)=0

Não entendi a pergunta. Seja f uma função bijetora e suave de R em R;
só podemos ter f'(x) = 0 se f''(x) também for 0.
Se f'(x) = f''(x) = f^(3)(x) = 0 então f^(4)(x) = 0
e mais geralmente toda raiz de f' é raiz de ordem par.
Era isso que você estava perguntando?

Aliás a única coisa que sabemos é que f'(x) >= 0 para todo x
ou f'(x) <= 0 para todo x.

[]s, N.
=
Instruções para entrar na lista, sair da lista e usar a lista em
http://www.mat.puc-rio.br/~nicolau/olimp/obm-l.html
O administrador desta lista é <[EMAIL PROTECTED]>
=



[obm-l] Re: [obm-l] Confirmação Probabilidade.

2002-10-16 Por tôpico Nicolau C. Saldanha

On Wed, Oct 16, 2002 at 08:44:38AM -0300, Hely Jr. wrote:
> Nesta Afirmação:
> 
> "Se existem 10 turmas e um aluno pertence a uma delas. A probabilidade do aluno 
>pertencer a uma das turmas é 1/10."
> 
> Esta resposta é falsa. Como relacionaria o evento aluno ao espaço amostral turma.

Não entendi nada, o que é hipótese, o que é pergunta,... 
Assim fica difícil ajudar. []s, N.
=
Instruções para entrar na lista, sair da lista e usar a lista em
http://www.mat.puc-rio.br/~nicolau/olimp/obm-l.html
O administrador desta lista é <[EMAIL PROTECTED]>
=



Re: [obm-l] bolinhas

2002-10-23 Por tôpico Nicolau C. Saldanha
On Wed, Oct 23, 2002 at 07:21:55AM -0200, Nicolau C. Saldanha wrote:
> On Tue, Oct 22, 2002 at 09:05:03PM -0200, Josimar wrote:
> > Olá companheiros da lista!
> > Um amigo meu me passou o problema que segue abaixo. Pensei um pouco e não
> > saiu, mas ele insistiu (disse que são três soluções, mas ele desconhece
> > todas) e eu lhe disse que colocaria o problema nesta lista e logo teria um
> > caminhão de respostas.  Aí vai: Imagine 12 bolinhas, sendo que 11 delas são
> > idênticas e 1 delas é difeituosa (ou mais leve, ou mais pesada). Como
> > descobrir qual é a bolinha defeituosa, com somente 3 pesagens?  []s, Josimar
> > 
> 
> Este é o conhecido problema das pesagens. Já foi discutido em uma Eureka
> e eu já mandei solução para esta lista. []s, N.

Mais precisamente...

Este é um caso particular do problema proposto número 10 que aparece
na página 59 da Eureka número 2. A solução do caso particular aparece
na Eureka número 4, página 54. A solução do caso geral aparece na
Eureka número 6, página 42 (claro!).

O problema também foi discutido nesta lista em agosto deste ano, veja,
entre outras,
http://www.mat.puc-rio.br/~nicolau/olimp/obm-l.200208/msg00213.html

[]s, N.

=
Instruções para entrar na lista, sair da lista e usar a lista em
http://www.mat.puc-rio.br/~nicolau/olimp/obm-l.html
O administrador desta lista é <[EMAIL PROTECTED]>
=



Re: [obm-l] bolinhas

2002-10-23 Por tôpico Nicolau C. Saldanha
On Wed, Oct 23, 2002 at 11:01:46AM -0200, Wagner wrote:
> Oi para todos!
> 
> Esse problema é famoso e tem resposta para 3^n bolinhas em que n é o nº de
> pesagens (mas ai é só uma resposta).  Nesse problema se usa uma balança de 2
> pratos e é preciso saber se a bolinha  é mais leve ou mais pesada,
> vou considerar que seja mais pesada.  1ª

Isto não é correto para o problema enunciado.

Há uma confusão aqui com um *outro* problema clássico (bem mais fácil)
em que sabemos a priori que a moeda (ou bolinha) diferente é
mais pesada (ou leve) que as outras. O problema enunciado *não* é este.

Aliás eu já mandei duas mensagens para a lista indicando onde o problema
originalmente proposto (o mais difícil) está discutido, só estou mandando
esta terceira para tentar esclarecer o mal entendido.

[]s, N.

=
Instruções para entrar na lista, sair da lista e usar a lista em
http://www.mat.puc-rio.br/~nicolau/olimp/obm-l.html
O administrador desta lista é <[EMAIL PROTECTED]>
=



Re: [obm-l] OBM-u

2002-10-23 Por tôpico Nicolau C. Saldanha
On Tue, Oct 22, 2002 at 03:34:44PM -0300, Eduardo Casagrande Stabel wrote:
> Eram essas as questões.
> 
> Questão 1.
> O gráfico de uma função polinomial de 4o. grau é cortada por uma reta em
> quatro pontos. Mostre que existe uma reta que corta esse gráfico em 4 pontos
> igualmente espaçados.
> 
> Questão 2.
> Uma matriz quadrada n por n tem diagonal por formada por 1s e as somas dos
> módulos dos elementos de cada linha não é maior do que 2. Mostre que o
> determinante está entre 0 (inclusive) e 1, não podendo ser 1.

O enunciado aqui deveria dizer que a matriz A é simétrica.
Não tenho a prova na mão mas o Gugu me diz que verificou o enunciado
e que a palavra 'simétrica' está lá sim.
> 
> Questão 3.
> Sejam A_1, A_2, ..., A_k subconjuntos de {1,2,3,...,n} satisfazendo |A_i| >=
> n/2 e |A_i \interseção A_j| <= n/4. Prove que |\união A_i| >= n. k / (k+1)
> 
> Questão 4.
> Resolva x = sqrt(2 + sqrt(2 - sqrt(2 + x))).
> 
> Questão 5.
> Define-se ln_0(x) = ln(x) e ln_(k+1)(x) = ln(ln_k(x))
> Dado n inteiro positivo seja k(n) o maior k tal que ln_k(n) >= 1.
> Defina a(n) = produtorio( ln_k(n), onde k varia de 0 até k(n) ).
> A série somatorio(1/a(n)) diverge ou converge?
> 
> Questão 6.
> Duas elipses, no plano, se cortam em quatro pontos.
> Prove que as oito retas tangentes nesses pontos às duas elipses tangenciam
> uma elipse ou uma circunferência.
> 
> Abraço,
> Eduardo.
> 
> From: "Paulo Santa Rita" <[EMAIL PROTECTED]>
> >
> > Ola Marcio e demais
> > colegas desta lista ... OBM-L,
> >
> > A questao e mesmo assim ? Nao entendi. Talvez eu tenha recebido a mensagem
> > truncada. Da pra enuncia-la ( bem como a questao 6 a que voce tambem se
> > referiu ) tal como apareceu na prova ?
> >
> > Um Abraco
> > Paulo Santa Rita
> > 3,1422,221002
> >
> > >Ateh agora nao consegui entender o enunciado da questao 5 direito.. Muito
> > >estranho aquilo.. como vc pode ter (ln...lnx) k(n) vezes se k(n) eh o
> maior
> > >inteiro k talque ln..ln(n) eh maior que 1? Dependendo do x, o ln...lnx
> pode
> > >nem mesmo estar definido..
> > >
> > >Mesmo que fosse n ao inves de x (dentro do produtorio), a questao parece
> > >ser bem dificil.. Alguma ideia?
> > >
> >
> >
> > _________
> > MSN Messenger: converse com os seus amigos online.
> > http://messenger.msn.com.br
> >
> > =
> > Instruções para entrar na lista, sair da lista e usar a lista em
> > http://www.mat.puc-rio.br/~nicolau/olimp/obm-l.html
> > O administrador desta lista é <[EMAIL PROTECTED]>
> > =
> >
> >
> 
> =
> Instruções para entrar na lista, sair da lista e usar a lista em
> http://www.mat.puc-rio.br/~nicolau/olimp/obm-l.html
> O administrador desta lista é <[EMAIL PROTECTED]>
> =
=
Instruções para entrar na lista, sair da lista e usar a lista em
http://www.mat.puc-rio.br/~nicolau/olimp/obm-l.html
O administrador desta lista é <[EMAIL PROTECTED]>
=



Re: [obm-l] bolinhas

2002-10-23 Por tôpico Nicolau C. Saldanha
On Tue, Oct 22, 2002 at 09:05:03PM -0200, Josimar wrote:
> Olá companheiros da lista!
> Um amigo meu me passou o problema que segue abaixo. Pensei um pouco e não
> saiu, mas ele insistiu (disse que são três soluções, mas ele desconhece
> todas) e eu lhe disse que colocaria o problema nesta lista e logo teria um
> caminhão de respostas.  Aí vai: Imagine 12 bolinhas, sendo que 11 delas são
> idênticas e 1 delas é difeituosa (ou mais leve, ou mais pesada). Como
> descobrir qual é a bolinha defeituosa, com somente 3 pesagens?  []s, Josimar
> 

Este é o conhecido problema das pesagens. Já foi discutido em uma Eureka
e eu já mandei solução para esta lista. []s, N.
=
Instruções para entrar na lista, sair da lista e usar a lista em
http://www.mat.puc-rio.br/~nicolau/olimp/obm-l.html
O administrador desta lista é <[EMAIL PROTECTED]>
=



Re: [obm-l] Grafos(novamente)

2002-10-25 Por tôpico Nicolau C. Saldanha
On Thu, Oct 24, 2002 at 05:56:22PM -0300, Vinicius José Fortuna wrote:
> Claro!
> 
> Mas o conceito está um pouco errado. Um grafo é hamiltoniano/euleriano se
> admite ciclo hamiltoniano/circuito euleriano, e não caminho
> hamiltoniano/trilha euleriana.

Não são estas as definições que eu considero usuais... []s, N.
=
Instruções para entrar na lista, sair da lista e usar a lista em
http://www.mat.puc-rio.br/~nicolau/olimp/obm-l.html
O administrador desta lista é <[EMAIL PROTECTED]>
=



Re: [obm-l] OBM-u

2002-10-22 Por tôpico Nicolau C. Saldanha
On Mon, Oct 21, 2002 at 10:05:39PM -0300, [EMAIL PROTECTED] wrote:
> Como que o pessoal aqui da lista foi na Olimpiada Universitaria? O que voces
> acharam da prova? 
> 
> Ateh agora nao consegui entender o enunciado da questao 5 direito.. Muito
> estranho aquilo.. como vc pode ter (ln...lnx) k(n) vezes se k(n) eh o maior
> inteiro k talque ln..ln(n) eh maior que 1? Dependendo do x, o ln...lnx pode
> nem mesmo estar definido..

Este enunciado infelizmente saiu errado.
Será publicado na home page da OBM um esclarecimento
e um pedido de desculpas em muito breve.
> 
> Mesmo que fosse n ao inves de x (dentro do produtorio), a questao parece ser
> bem dificil.. Alguma ideia?

Esta é a idéia. Sim, a questão é difícil.
> 
> Considerando a dificuldade em saber o enunciado da 5, e a minha incapacidade
> de pensar na 6 (e um arrependimento por nao ter estudado em casa as anotacoes
> da aula de geometria projetiva da semana olimpica :) ), pude dedicar umas 3hs
> da minha prova a questao 4 (resolver x=sqrt(2+sqrt(2-sqrt(2+x))).. Depois de
> ficar tentando fatorar o polinomio resultante de se elevar tudo ao quadrado
> diversas vezes, acabei tendo a sorte de fazer x=2cosy (engracado q foi a
> mesma coisa q eu usei na obmu do ano passado.. 1+cosy = 2cos^2(y/2)..)..
> 
> Mandem seus comentarios sobre a prova..
> 
> Abracos,
> Marcio
> 
> 
> 
> 
> 
> =
> Instruções para entrar na lista, sair da lista e usar a lista em
> http://www.mat.puc-rio.br/~nicolau/olimp/obm-l.html
> O administrador desta lista é <[EMAIL PROTECTED]>
> =
=====
Instruções para entrar na lista, sair da lista e usar a lista em
http://www.mat.puc-rio.br/~nicolau/olimp/obm-l.html
O administrador desta lista é <[EMAIL PROTECTED]>
=



Re: [obm-l] Grafos(novamente)

2002-10-24 Por tôpico Nicolau C. Saldanha
On Thu, Oct 24, 2002 at 04:48:31PM -0300, Carlos Maçaranduba wrote:
>  
> 
> 
>  Um grafo pode ser hamiltoniano e euleriano ao mesmo
>  tempo??Ou seja ter caminho hamiltoniano e caminho
>  euleriano ao mesmo tempo


Pode:

   *-*-*-*-*

é hamiltoniano e euleriano, como você não deve ter dificuldade em verificar.
Aliás o caminho hamiltoniano é também euleriano.
=
Instruções para entrar na lista, sair da lista e usar a lista em
http://www.mat.puc-rio.br/~nicolau/olimp/obm-l.html
O administrador desta lista é <[EMAIL PROTECTED]>
=



Re: [obm-l] E o Nivel Tres,ninguem faz nada??????

2002-10-25 Por tôpico Nicolau C. Saldanha
On Fri, Oct 25, 2002 at 04:26:31PM -0300, Guilherme Fujiwara wrote:
> Se interessar a alguém segue a minha solução da 6:
> .
> .
> .
> .
> Considere que há N seqüências no dicionário.
> Associe a cada seqüência todas as que estão a
> distância <= 3 dela e as que estão a distância 4 e
> diferem na primeira coordenada(no tamanho do primeiro
> sinal de fumaça). Basta verificar que cada elemento de
> {0;1}^24 foi contado no máximo uma vez e que a cada
> seqüência do dicionário foram associados exatamente:
> C(24;0)+C(24;1)+C(24;2)+C(24;3)+C(23;3)=4096 elementos
> de {0;1}^24, logo 4096.N<=2^24 <=> n<=4096
> A única coisa que eu errei foi essa última conta com
> os binomiais, e não percebi que tinha feito a questão
> (mas como eu sou estúpido), escrevi na prova mais como
> uma idéia.
> Issao.

Muito bom.

O mais interessante nesta questão (e que não é pedido na prova)
é exibir um dicionário com exatamente 4096 elementos.

[]s, N.
=
Instruções para entrar na lista, sair da lista e usar a lista em
http://www.mat.puc-rio.br/~nicolau/olimp/obm-l.html
O administrador desta lista é <[EMAIL PROTECTED]>
=



Re: [obm-l] universitaria

2002-10-17 Por tôpico Nicolau C. Saldanha
On Thu, Oct 17, 2002 at 09:46:43AM -0300, Eric Campos Bastos Guedes wrote:
> nao tenho recebido mensagens esses ultimos dias (desde o dia 6). Algum
> problema com a lista?

Houve um problema com a data no servidor.
Isto faz com que as mensagens apareçam em

http://www.mat.puc-rio.br/~nicolau/olimp/obm-l.html

fora da ordem correta. Mas as mensagens novas aparecem lá sim.
Se você não estiver recebendo escreva para mim (por fora da lista).

> Gostaria de saber onde farei a prova da segunda fase da OBMU. Moro em
> Niteroi e fiz a primeira fase na UFF.

Se a UFF aplicar a 2a fase você pode fazer aí mesmo.
Se a UFF não aplicar a 2a fase você pode vir fazer prova na PUC.

[]s, N.
=
Instruções para entrar na lista, sair da lista e usar a lista em
http://www.mat.puc-rio.br/~nicolau/olimp/obm-l.html
O administrador desta lista é <[EMAIL PROTECTED]>
=



Re: [obm-l] E o Nivel Tres,ninguem faz nada??????

2002-10-28 Por tôpico Nicolau C. Saldanha
On Sat, Oct 26, 2002 at 08:59:12PM -0300, Murilo Andrade wrote:
> Olá,
> Eu dei exatamente este exemplo na minha solução. Outro
> 
> exemplo que eu citei foi o trivial A =
> {2003,2003,2003,
> ...,2003} (ou outro primo qualquer maior que 2002).
> Será que aceitam que existam elementos iguais no
> conjunto?

Certamente que não, isso vai totalmente contra o conceito de conjunto.
[]s, N.
=
Instruções para entrar na lista, sair da lista e usar a lista em
http://www.mat.puc-rio.br/~nicolau/olimp/obm-l.html
O administrador desta lista é <[EMAIL PROTECTED]>
=



[obm-l] Re: [obm-l] cultura inútil

2002-10-28 Por tôpico Nicolau C. Saldanha
On Sun, Oct 27, 2002 at 06:42:15PM -0200, Jose Francisco Guimaraes Costa wrote:
> Junto com a edição de domingo do Jornal do Brasil, do Rio, circula a revista
> Domingo, cuja seção Listas da Domingo relaciona a cada semana 10 itens
> co-relacionados.
> 
> A Listas da Domingo do domingo passado tinha como título "Dez Coisas que
> Aprendemos na Escola... para Nunca Mais", e como sub-título "Ser aluno até os
> anos 70 era padecer num mar de decorebas inúteis ou de detalhes sem maior
> importância". Entre as decorebas inúteis e detalhes sem importância (o que
> vem entre parênteses é a explicação do autor da lista):
> 
> - O nome completo de D. Pedro I (Pedro de Alcântara Francisco Antonio Miguel
> Rafael...)
> - Os afluentes do Amazonas (Juruá, Madeira, Xingu, Tapajós...)
> - Os donos das Capitanias Hereditárias (Martin Afonso, Duarte Coelho, Pero de
> Góes...)
> - A Tabela Periódica dos Elementos (Hidrogênio, Hélio, Lítio...) e...
> - Os números imaginários (eram o resultado da raiz quadrada de
> um número negativo) 
> 
> Reparem no tempo do verbo na explicação do que são números imaginários:
> "eram" - quer dizer, números imaginários _não são mais_ o resultado da raiz
> quadrada de um número negativo.

Parece que o objetivo da reportagem era mostrar defeitos na educação brasileira.
Isto o autor conseguiu. Incrivelmente bem, se você entendem o que quero dizer.
;-)

Mais seriamente, acho que esta matéria, mesmo que escrita por um idiota
que não sabe do que está falando, pode nos dar material para refletir.
Para demonstrar que números complexos devem ser ensinados no ensino médio,
não basta argumentar que números complexos sejam importantes para um monte
de coisa que o aluno *não sabe*. É preciso que o aluno seja capaz de apreciar
*alguma* utilidade para o que ele está aprendendo naquele momento. Estas
aplicações elementares de números complexos são do conhecimento de todos
os leitores desta lista mas são muito distantes do comum do ensino médio.

O mesmo vale aliás para vários outros tópicos do currículo escolar,
como números primos (para crianças menores) e matrizes. Os alunos
aprendem aquelas coisas sem entender para que servem (talvez muitos
professores também não saibam) e esquecem tudo.

[]s, N.
=====
Instruções para entrar na lista, sair da lista e usar a lista em
http://www.mat.puc-rio.br/~nicolau/olimp/obm-l.html
O administrador desta lista é <[EMAIL PROTECTED]>
=



[obm-l] Re: [obm-l] cultura inútil

2002-10-28 Por tôpico Nicolau C. Saldanha
On Sun, Oct 27, 2002 at 03:22:41PM -0500, niski wrote:
> >
> > - Os números imaginários (eram o resultado da raiz quadrada de um 
> > número negativo)
> >  
> > Reparem no tempo do verbo na explicação do que são números 
> > imaginários: "eram" - quer dizer, números imaginários _não são mais_ o 
> > resultado da raiz quadrada de um número negativo.
> 
> 
> 
>   Mas nunca foram, já que a radiciação para números reais é definida 
> para radicandos maiores ou iguais a 0. Ou seja o simbolo sqrt(-1) não 
> existe.

É um equívoco ontológico dizer que um *símbolo* não existe.
O símbolo é uma mancha de tinta no papel (ou giz no quadro,
ou uma seqüência de bytes) e claramente existe.

A função raiz quadrada é definida com domínio [0,+infinito) enquanto
operamos apenas com números reais. Quando operamos com números complexos
o domínio aumenta, pode ser tomado como todo o plano complexo desde
que definamos um corte. Este aumento de domínio é algo comum em matemática.
A pergunta "existe raiz quadrada de -1?" deve receber respostas diferentes
conforme o contexto (ou deve-se pedir um esclarecimento quanto a qual
a classe de números com a qual estamos trabalhando).

[]s, N.


=
Instruções para entrar na lista, sair da lista e usar a lista em
http://www.mat.puc-rio.br/~nicolau/olimp/obm-l.html
O administrador desta lista é <[EMAIL PROTECTED]>
=



[obm-l] Re: [obm-l] onde está o erro?

2002-10-29 Por tôpico Nicolau C. Saldanha
On Tue, Oct 29, 2002 at 02:31:24AM -0300, cgmat wrote:
>  
> Onde está o erro?
> 
> Seja S a soma dos termos infinitos de uma PG de números estritamente positivos
> com razão 2 e a1=1.
> 
> S = (1 + 2 + 4 + 8 + 16 + 32 + ...) => a partir do a2, todos os termos são múltiplos 
>de 2.
> 
> Se colocarmos o 2 em evidência, teremos:
> 
> S = 1 + 2 . ( 1 + 2 + 4 + 8 + 16 + 32 + ... ) => como S = ( 1 + 2 + 4 + 8 + 16 + 32 
>+ ... ), temos:
> 
> S = 1 + 2.S
> S - 2.S = 1
> grato, cgomes.
> 

O erro está em supor que a soma

S = (1 + 2 + 4 + 8 + 16 + 32 + ...)

faz sentido. A definição de uma soma infinita envolve limites e o limite
pode existir ou não; no caso não existe. Veja um bom livro de análise para
a definição de limite e outras considerações teóricas.

Em certo sentido, entretanto, não há nada errado: S = -1.
Estudam-se somas de séries divergentes e este é um dos exemplos mais simples.
Há por exemplo um livro do Hardy (o mesmo que escreveu o clássico
de teoria dos números) sobre este assunto. Mas tome cuidado para
não se empolgar demais com somas de séries divergentes e sair tirando
conclusões absurdas.

[]s, N.
=
Instruções para entrar na lista, sair da lista e usar a lista em
http://www.mat.puc-rio.br/~nicolau/olimp/obm-l.html
O administrador desta lista é <[EMAIL PROTECTED]>
=



Re: [obm-l] OBM-u(e essa tal elipse?)(; ;)

2002-10-30 Por tôpico Nicolau C. Saldanha
On Tue, Oct 29, 2002 at 01:55:31PM -0300, Johann Peter Gustav Lejeune Dirichlet wrote:
> 
> Alo Shine,tudo blz?Bem,eu tava pensando nessa ideia de projetar,mas e essa 
>transformaçao de elipse no eixo da circunferencia(ou o contrario?)?
> Te maisAss.:Johann 
>  Carlos Yuzo Shine <[EMAIL PROTECTED]> wrote:Oi Humberto e demais amigos da lista!!
> 
> Tudo bem?
> 
> Puxa, eu tive a idéia de considerar o produto dos
> auto-valores também, mas como demorei muito no caso
> n=3 (pensei demais nos casos pequenos...), acabei não
> tendo tempo para finalizar a idéia... eu pensei na
> existência de um auto-vetor positivo, mas acabei não
> conseguindo nem ter tempo para pensar nessa parte do
> problema.
> 
> No 6 eu projetei uma das elipses numa curcunferência.
> Mas o que não sabia era que dava para fazer uma
> transformação de modo que as elipses virem uma
> circunferência e uma elipse cuja reta suporte de um
> eixo passa pelo centro da circunferência. Aí ficava
> mais fácil. Mas, pelo que soube, existe uma solução
> projetiva (né Luciano? ;) ).

Minha solução é a seguinte:

Existe uma transformação projetiva que leva uma elipse no círculo
unitário (aliás basta pegar uma translação seguida de uma transformação
linear). Depois disso existe uma outra transformação projetiva
que mantem o círculo unitário e leva os quatro pontos de interseção
nos vértices de um retângulo com os lados paralelos aos eixos
(de fato, transformações projetivas que respeitam o círculo unitário
funcionam como transformações de Möbius no círculo unitário devidamente
identificado com R pela projeção estereográfica, que aliás também é Möbius).
Com isso as duas elipses são da forma x^2/a^2 + y^2/b^2 = 1.
Agora o problema fica fácil.

Eu mostrei esta solução para o Luciano mas ele acabou não me mostrando
a dele (parece que era mais longa).

[]s, N.
=
Instruções para entrar na lista, sair da lista e usar a lista em
http://www.mat.puc-rio.br/~nicolau/olimp/obm-l.html
O administrador desta lista é <[EMAIL PROTECTED]>
=



Re: [obm-l] Nao entendi a solucao do Stein...

2002-10-30 Por tôpico Nicolau C. Saldanha
On Tue, Oct 29, 2002 at 05:37:21PM -0300, Carlos Gustavo Tamm de Araujo Moreira wrote:
>Pera ai,nao entendi:
>C(24;0)+C(24;1)+C(24;2)+C(24;3)+C(23;3)=2325,e nao 4096...
>    O que o Nicolau quer dizer com "Muito bom"?
>Abracos,
>Gugu

Oi Gugu, acho que você errou as contas, veja o que o maple diz:

> binomial(24,0);
   1

> binomial(24,1);
  24

> binomial(24,2);
  276

> binomial(24,3);
 2024

> binomial(23,3);
 1771

> 1+24+276+2024+1771;
 4096

=
Instruções para entrar na lista, sair da lista e usar a lista em
http://www.mat.puc-rio.br/~nicolau/olimp/obm-l.html
O administrador desta lista é <[EMAIL PROTECTED]>
=



Re: [obm-l] Paradoxo

2002-10-31 Por tôpico Nicolau C. Saldanha
On Thu, Oct 17, 2002 at 05:33:42PM -0300, Wagner wrote:
> Oi para todos!
> 
> Quero saber se a afirmação abaixo é ou não um paradoxo:
> 
> --->  p = pi
> 
> Seja x^2p = a^2p.  Pelo teorema de De Moivre : x = a.(i.sen k + cos k)  para
> k inteiro.

Pq k seria inteiro?

> Seja n(k) o número de valores possíveis de k, tais que se k1 é
> diferente de k2, x1 é diferente de x2.

Não existe tal número n(k).

> Representando graficamente os valores
> de x, quando o número de valores de x tende a n(k), os pontos que representam
> os valores de x tendem a uma circunferência de raio a. Então isso quer dizer
> que se z é um número complexo qualquer, z^2p = |z|^2p, ou seja qualquer
> número complexo elevado a 2p é um número real.

Como n(k) não existe, isso não prova nada.
Ou sob um ponto de vista mais otimista, podemos inverter o raciocínio
e ver isso como uma demonstração de que não existe n(k).

[]s, N.
=
Instruções para entrar na lista, sair da lista e usar a lista em
http://www.mat.puc-rio.br/~nicolau/olimp/obm-l.html
O administrador desta lista é <[EMAIL PROTECTED]>
=



Re: [obm-l] OBM-u(e essa tal elipse?)(; ;)

2002-11-01 Por tôpico Nicolau C. Saldanha
On Wed, Oct 30, 2002 at 04:07:25PM -0200, Marcio wrote:
> O que exatamente significa uma transformacao projetiva? Na prova eu
> cheguei a escrever que era possivel, via uma transformacao linear,
> considerar o problema "mais simples" no qual uma das elipses eh um circulo..
> Mas nao sabia que era possivel reduzir ao caso em que os eixos da elipse que
> sobra eram paralelos aos eixos x,y (e pelo jeito, concentrica com o
> circulo)..  Tem alguma referencia legal onde eu possa saber mais sobre
> transformacoes desse tipo? Esse parece ser um assunto interessante para a
> proxima semana olimpica...

Identifique o plano xy com o plano z=1 em R^3. Considere uma transformação
linear (ou matriz) 3x3 A qq que leva (x,y,1) em (u,v,w). Bem, o ponto (u,v,w)
em geral não está no nosso plano mas podemos projetá-lo lá radialmente
multiplicando por 1/w: assim a imagem de (x,y,1) será (u/w,v/w,1).
Eliminando a terceira coordenada, uma transformação projetiva é

(x,y) |-> ((a11 x + a12 y + a13)/(a31 x + a32 y + a33),
   (a21 x + a22 y + a23)/(a31 x + a32 y + a33))

com det A diferente de zero. Estas transformações levam cônicas em cônicas.

[]s, N.
=
Instruções para entrar na lista, sair da lista e usar a lista em
http://www.mat.puc-rio.br/~nicolau/olimp/obm-l.html
O administrador desta lista é <[EMAIL PROTECTED]>
=



Re: [obm-l] Maple

2002-11-01 Por tôpico Nicolau C. Saldanha
On Fri, Nov 01, 2002 at 05:23:02PM -0200, Gabriel Pérgola wrote:
> Onde posso baixar o maple completo?

O maple é um programa comercial, você não pode baixá-lo gratuitamente.
[]s, N.
=
Instruções para entrar na lista, sair da lista e usar a lista em
http://www.mat.puc-rio.br/~nicolau/olimp/obm-l.html
O administrador desta lista é <[EMAIL PROTECTED]>
=



Re: [obm-l] Maple

2002-11-03 Por tôpico Nicolau C. Saldanha
On Fri, Nov 01, 2002 at 11:25:06PM -0200, Diego wrote:
> Existe um programa bonzinho que faz muitas coisas que o Maple faz, e é
> freeware.
> 
> É o MuPAD. Você pode baixá-lo de www.mupad.de

Ou melhor, mupad também é comercial e pago mas existe uma versão que
pode ser usada gratuitamente (e legalmente). Existem alguns programas
realmente livres (GPL ou similar) que fazem algumas das coisas que o maple
faz mas aí depende dos seus interesses.

[]s, N.
=
Instruções para entrar na lista, sair da lista e usar a lista em
http://www.mat.puc-rio.br/~nicolau/olimp/obm-l.html
O administrador desta lista é <[EMAIL PROTECTED]>
=



Re: [obm-l] desafio

2002-11-03 Por tôpico Nicolau C. Saldanha
On Sat, Nov 02, 2002 at 01:25:13AM -0200, Augusto César Morgado wrote:
> Não. Ninguém é capaz de provar isso, nem usando nem não usando..
> Morgado
> 
> glauber.morais wrote:
> 
> >Olá,
> >   
> >Alguém seria capaz de provar o seguinte lim sem 
> >utilizar o lim fundamental do sen:
> >
> >   lim n.tg(n/x)=n
> >   x->inf
> > 
> > ou
> >
> >  lim n.sen(n/x)=n
> >  x->inf  

Para quem não entendeu o sarcasmo do Morgado, estes limites
estão errados (talvez haja um erro tipográfico). Os dois limites
são iguais a 0 por motivos triviais (continuidade) sem ter nada
a ver com o limite fundamental.

[]s, N.
=
Instruções para entrar na lista, sair da lista e usar a lista em
http://www.mat.puc-rio.br/~nicolau/olimp/obm-l.html
O administrador desta lista é <[EMAIL PROTECTED]>
=



Re: [obm-l] Gnu-Pari

2002-11-03 Por tôpico Nicolau C. Saldanha
On Sat, Nov 02, 2002 at 06:11:15PM +, Fernanda Medeiros wrote:
> 
> 
> 
> Alguem pode me dizer como funciona o gnu pari? Eu "baixei" o pari.tgz(?) e o 
> pari-alpha.tgz (?) (usei para isso o site que o prof. Tengan deu na semana 
> olímpica) mas não estou vendo como usá-lo... é necessário algum programa 
> especial?
> Valeu
> FÊ

Eu não sei o que é pari mas um arquivo *.tgz é análogo a um arquivo *.zip:
uma forma de guardar um monte de coisa em um arquivo só. Você precisa
antes de mais nada ver o que tem dentro. Se você estivesse usando [Gnu+]Linux
você saberia disso; se você estiver usando Windows o winzip deve ser capaz
de abrir os seus arquivos. Mas note que sendo Gnu, o pari deve ser feito
principalmente para [Gnu+]Linux...

[]s, N.
=
Instruções para entrar na lista, sair da lista e usar a lista em
http://www.mat.puc-rio.br/~nicolau/olimp/obm-l.html
O administrador desta lista é <[EMAIL PROTECTED]>
=



Re: [obm-l] Somatorio(numeros complexos)

2002-11-03 Por tôpico Nicolau C. Saldanha
On Sat, Nov 02, 2002 at 07:52:26PM +, leonardo mattos wrote:
> Ola,
> Alguem poderia resolver essa questao pra mim por numeros complexos?!
> 
> S=1+cos(x)+cos(2x)+...cos(nx)  e  S´=1+sen(x)+sen(2x)+...+sen(nx)

Chame S'' = S' - 1 = 0 + sen(x) + ... + sen(nx).

Temos S + i S'' = 1 + z + z^2 + ... + z^n onde z = cos(x) + i sen(x).
Some agora esta PG da forma usual...

[]s, N.
=
Instruções para entrar na lista, sair da lista e usar a lista em
http://www.mat.puc-rio.br/~nicolau/olimp/obm-l.html
O administrador desta lista é <[EMAIL PROTECTED]>
=



[obm-l] Re: [obm-l] Os números de 2 caras

2002-11-03 Por tôpico Nicolau C. Saldanha
On Fri, Nov 01, 2002 at 09:03:42PM -0200, Wagner wrote:
> Oi para todos!
> 
> Esse é um problema bem simples e interessante:
> 
> Seja a um número complexo qualquer (ou seja pertencente a C).
> Para que valores de a temos que:
> (-a)^k=a^(-k), para todo k real ?

É bom lembrar que z^x (com z complexo e x real) não é uma coisa
bem definida, depende de ramos. Por exemplo, quanto vale (-i)^(1/2)?
Se escrevermos -i = exp(-i Pi/2) teremos
(-i)^(1/2) = a - i a onde a = sqrt(2)/2.
Mas se escrevermos -i = exp(i 3 Pi/2) teremos (-i)^(1/2) = - a + i a.
E os dois valores não são iguais.

[]s, N.

=
Instruções para entrar na lista, sair da lista e usar a lista em
http://www.mat.puc-rio.br/~nicolau/olimp/obm-l.html
O administrador desta lista é <[EMAIL PROTECTED]>
=



[obm-l] Re: [obm-l] Re: Os números de 2 caras

2002-11-03 Por tôpico Nicolau C. Saldanha
On Sat, Nov 02, 2002 at 06:01:40PM -0300, Erasmo de Souza Dias wrote:
> 
> Realmente é um problema interessante e aqui vai umas das soluções.

Eu achei o problema um pouco problemático por causa da mal-definição
de z^x, z complexo e x real, mas...
 
> Uma solução trivial para (-a)k = a-k , é a=0 ,claro

... a = 0 não é uma solução correta.
0^x = 0 apenas para x real estritament positivo; 0^0 = 1;
para outros valores de x (reais negativos ou complexos)
0^x não está definido.

[]s, N.
=
Instruções para entrar na lista, sair da lista e usar a lista em
http://www.mat.puc-rio.br/~nicolau/olimp/obm-l.html
O administrador desta lista é <[EMAIL PROTECTED]>
=



Re: [obm-l] Ajuda em probabilidade

2002-11-04 Por tôpico Nicolau C. Saldanha
On Mon, Nov 04, 2002 at 12:09:38AM -0200, Felipe Villela Dias wrote:
> Um moeda é viciada, ou seja tem uma probabilidade p, p diferente de 50%, de
> dar cara e uma probabilidade 1 - p de dar coroa. Sendo assim, se você jogar a
> moeda infinitas qual a probabilidade de que em pelo menos um instante o
> número de vezes que saiu cara vai ser igual ao número de vezes que saiu
> coroa?

Não vai dar para dar uma demonstração totalmente rigorosa mas acho
que isso deve deixar você satisfeito.

Suponha p < 1/2, assim cara é menos provável e a longo prazo teremos
mais coroas do que caras. Supondo que em um certo instante as coroas
tenham uma vantagem n, seja f(n) a probabilidade de que naquele instante
ou mais tarde venhamos a ter um empate. Se n = 0 temos f(n) = 1
por definição. Se n < 0 também temos f(n) = 1 pois as coroas inevitavelmente
superarão qualquer desvantagem. O difícil é saber quanto vale f(n) para
n > 0. Observe que claramente temos 0 < f(n) < 1 para todo n > 0
e temos também lim_{n -> infinito} f(n) = 0. Além disso, para todo n > 0
temos 

f(n) = p f(n-1) + (1-p) f(n+1)

pois jogando uma vez a moeda reduzimos o problema ao caso n-1 ou n+1
conforme sair cara ou coroa, respectivamente. É natural agora conjecturar
(e fácil provar) que

f(n) = a^n para n >= 0 onde a = p/(1-p).

A resposta para o seu problema não é f(0) (que é 1 por definição)
pois aí não contamos o empate inicial de 0x0 antes do jogo começar.
A resposta é portanto

p f(-1) + (1-p) f(1) = p + p = 2p.

No caso p > 1/2 basta trocar todos os p por 1-p e a resposta é 2(1-p).

[]s, N.

=
Instruções para entrar na lista, sair da lista e usar a lista em
http://www.mat.puc-rio.br/~nicolau/olimp/obm-l.html
O administrador desta lista é <[EMAIL PROTECTED]>
=



Re: [obm-l] Ajuda em probabilidade

2002-11-04 Por tôpico Nicolau C. Saldanha
On Mon, Nov 04, 2002 at 08:23:48AM -0200, Augusto César Morgado wrote:
> O problema é complicado, no sentido que exige um conhecimento específico 
> de algumas técnicas de probabilidade. Veja o livro do Feller (capítulo 
> 11), na parte de Passeios Aleatórios e procure por Retorno À Origem.
> A propósito, a resposta é 1 - módulo (2p-1)
> 
> Felipe Villela Dias wrote:
> 
> > Um moeda é viciada, ou seja tem uma probabilidade p, p diferente de 
> > 50%, de dar cara e uma probabilidade 1 - p de dar coroa. Sendo assim, 
> > se você jogar a moeda infinitas qual a probabilidade de que em pelo 
> > menos um instante o número de vezes que saiu cara vai ser igual ao 
> > número de vezes que saiu coroa?

Mandei uma solução (ou pelo menos esboço de solução) em outra mensagem.
Só queria comentar que a fórmula do Morgado coincide com a resposta
que eu obtive:

1 - |2p - 1| = 2p, p <= 1/2
1 - |2p - 1| = 2(1-p), p >= 1/2

[]s, N.
=
Instruções para entrar na lista, sair da lista e usar a lista em
http://www.mat.puc-rio.br/~nicolau/olimp/obm-l.html
O administrador desta lista é <[EMAIL PROTECTED]>
=



Re: [obm-l] Problemas

2002-11-04 Por tôpico Nicolau C. Saldanha
On Thu, Jan 01, 1998 at 04:14:42AM -0200, Fernando wrote:
> 
> 
> Gostaria de ajuda para solucionar as seguintes questões:
> 1) Achar os valores inteiros e positivos de n para os quais o trinômio n^2 + n + 43 
>é um quadrado.

Escreva 

n^2 + n + 43 = m^2

Completando quadrados,

(n + 1/2)^2 + 171/4 = m^2

m^2 - (n + 1/2)^2 = 171/4

(2m + 2n + 1)(2m - 2n - 1) = 171

Agora basta considerar as possíveis fatorações de 171 = 3*3*19:

2m + 2n + 1 = 171; 2m - 2n - 1 = 1  ->  n = 42
2m + 2n + 1 = 57;  2m - 2n - 1 = 3  ->  n = 13
2m + 2n + 1 = 19;  2m - 2n - 1 = 9  ->  n = 2

Verificando,

42^2 + 42 + 43 = 43^2
13^2 + 13 + 43 = 15^2
2^2 + 2 + 43 = 7^2

[]s, N.

=
Instruções para entrar na lista, sair da lista e usar a lista em
http://www.mat.puc-rio.br/~nicolau/olimp/obm-l.html
O administrador desta lista é <[EMAIL PROTECTED]>
=



[obm-l] Re: [obm-l] Re: [obm-l] Re: [obm-l] Re: Os números de 2 caras

2002-11-05 Por tôpico Nicolau C. Saldanha
On Tue, Nov 05, 2002 at 05:20:32PM -0200, Wagner wrote:
> Oi pessoal !
> 
> 0^0 não existe, mas lim (x--> 0) x/x = 1
> (Assim como outros limites de f(x) quando f(x)=0/0)
> De qualquer forma valeu pelo lembrete.

O usual é definir 0^0 = 1 sim. Existem várias razões para isso.
Aliás este assunto já foi discutido nesta lista, procure nos arquivos.

[]s, N.

=
Instruções para entrar na lista, sair da lista e usar a lista em
http://www.mat.puc-rio.br/~nicolau/olimp/obm-l.html
O administrador desta lista é <[EMAIL PROTECTED]>
=



Re: [obm-l] Ajuda em Teorema do Numero primo e Teorema de Dirichlet

2002-11-06 Por tôpico Nicolau C. Saldanha
On Wed, Nov 06, 2002 at 12:53:19PM -0300, Johann Peter Gustav Lejeune Dirichlet
wrote:
> 
> Turma,andei fazendo uns passeios pela USP e pesquisando sobre o TNP.Acabei
> caindo no Teorema da PA de Dirichlet(se a razao de uma PA e prima com seu
> primeiro termo,entao a dita PA contera infinitos primos).Tudo saia dessa
> desigualdade aqui:
> 
> Sp<=x ((log p)/p)=log x+0(1),em que p significa "primo".
> 
> Alguem sabe como demonstrar?

Não entendi nada. O que é esse S?

Eu conheço uma demonstração do teorema de Dirichlet usando variável complexa
e funções tipo zeta, tem no livro do Borevich-Shafarevich. Conheço uma
demonstração elementar no caso em que o primeiro termo da PA é 1.

Teorema: Seja n um inteiro positivo dado;
existem infinitos primos da forma p = nk + 1.

Esboço de dem:

Seja P(x) o polinômio mônico cujas raízes são as raízes primitivas de ordem
n da unidade, i.e., P(x) = (x-z1)...(x-zm) onde z1, ..., zm são os números
complexos que satisfazem z^n = 1 e z^m = 1 -> n|m.
Não é difícil mostrar que este polinômio tem coeficientes inteiros.

Sejam p1, p2, ..., pm primos da forma nk+1.
Considere N = P(n! * p1 * ... * pm).
Claramente p1, ..., pm, assim como primos divisores de n
não podem ser divisores de N. Por outro lado não é difícil provar
que se q é um fator primo de N então q é da forma nk+1.
Assim um fator primo de N é um novo primo da forma nk+1.
=
Instruções para entrar na lista, sair da lista e usar a lista em
http://www.mat.puc-rio.br/~nicolau/olimp/obm-l.html
O administrador desta lista é <[EMAIL PROTECTED]>
=



[obm-l] Re: [obm-l](correçao) Ajuda em Teorema do Numero primo e Teorema de Dirichlet

2002-11-07 Por tôpico Nicolau C. Saldanha
On Thu, Nov 07, 2002 at 12:21:06PM -0300, Johann Peter Gustav Lejeune Dirichlet wrote:
> 
> Ah,me lembrei.Esse Sp<=x significa somatorio sobre todos os primos que nao passam de 
>x,e esse 0(1) e aquele treco de Landau,se nao me engano
>  "Nicolau C. Saldanha" <[EMAIL PROTECTED]> wrote:On Wed, Nov 06, 2002 at 
>12:53:19PM -0300, Johann Peter Gustav Lejeune Dirichlet
> wrote:
> > 
> > Turma,andei fazendo uns passeios pela USP e pesquisando sobre o TNP.Acabei
> > caindo no Teorema da PA de Dirichlet(se a razao de uma PA e prima com seu
> > primeiro termo,entao a dita PA contera infinitos primos).Tudo saia dessa
> > desigualdade aqui:
> > 
> > Sp<=x ((log p)/p)=log x+0(1),em que p significa "primo".
> > 
> > Alguem sabe como demonstrar?

Agora entendi. Mas é difícil, isso é quase o TNP
(ou pelo menos uma versão fraca dele). []s, N.
=====
Instruções para entrar na lista, sair da lista e usar a lista em
http://www.mat.puc-rio.br/~nicolau/olimp/obm-l.html
O administrador desta lista é <[EMAIL PROTECTED]>
=



Re: [obm-l] Probabilidade

2002-11-08 Por tôpico Nicolau C. Saldanha
On Fri, Nov 08, 2002 at 11:47:09AM -0300, Wendel Scardua wrote:
> 
> > Escolhe-se ao acaso três vértices distintos de um cubo.
> > A probabilidade de que estes vértices pertençam a uma mesma face é?
> 
> Deixa eu ver...
> Se um vértice foi escolhido, a chance do segundo pertencer a mesma face
> seria 6/7 (o único que não serve é o oposto ao primeiro)
> Agora a chance de escolher o terceiro seria 2/6 (os únicos que servem
> são os que completam a face...
> Então a probabilidade dos três vértices pertencerem a mesma face deve ser
>  6/7 * 2/6 == 2/7
> 
> (A não ser que eu tenha algum erro bobo... ^ ^")

Infelizmente há um erro sim. Ao escolher o segundo ponto ele pode
estar em uma mesma aresta que o ponto inicial;
neste caso há 4 (e não 2) possibilidades para o terceiro ponto.
Você pode remendar a solução considerando dois casos mas talvez
seja mais fácil fazer assim.

Quantas conjuntos de três vértices existem?

binomial(8,3) = 56  

Destes conjuntos, quantos estão contidos em uma face?

Se eles estiverem contidos em uma face, esta face é única pois
não há no cubo três vértices colineares. Para cada uma das 6 faces
há 4 possibilidades (basta escolher um ponto para ficar de fora).
Assim, existem 24 conjuntos contidos em uma face.

Resposta final: 24/56 = 3/7.

[]s, N.
=
Instruções para entrar na lista, sair da lista e usar a lista em
http://www.mat.puc-rio.br/~nicolau/olimp/obm-l.html
O administrador desta lista é <[EMAIL PROTECTED]>
=



Re: [obm-l] P e NP

2002-11-11 Por tôpico Nicolau C. Saldanha
On Sun, Nov 10, 2002 at 01:35:24PM -0300, Domingos Jr. wrote:
> Basicamente problemas da classe P são aqueles para os quais existe um
> algoritmo que determina a(s) solução(ões) em tempo polinomial, problemas NP
> são aqueles problemas considerados difíceis pois não existe solução
> polinomial, só exponencial.

Não é bem assim. NP não significa não-polinomial, significa
"nondeterministic polynomial". Ou seja, se você tiver sorte
(ou uma iluminação sobrenatural), o problema NP passa a ser polinomial.
Uma outra forma de dizer isso é que você deve poder *verificar*
uma solução (talvez surpreendente) que alguém te mostre em tempo
polinomial.

Os problemas verdadeiramente difíceis são exponenciais mesmo e se você
tiver sorte de "adivinhar" a resposta ainda é exponencialmente difícil
verificar a resposta.

> 
> Para exemplos de problemas NP-completo temos o caso do caxeiro viajante
> (muito famoso), problemas de grafos, otimização inteira etc... uma pequena
> busca na internet vai te retornar muitos links para esses assuntos.

O exemplo do caixeiro viajante é bom: determinar se existe um caminho
ou circuito hamiltoniano em um grafo é (ou pelo menos parece ser)
um problema difícil. Por outro lado, se alguém te mostrar um caminho
hamiltoniano é bem fácil verificar que o caminho é, de fato, hamiltoniano...

Um exemplo de problema NP é verificar que um grafo pode ser pintado
com três cores sem que vértices da mesma cor fiquem sendo vizinhos:
se você adivinhar a resposta é fácil verificar que ela está certa.
Um exemplo de problema realmente difícil (não NP) é verificar
que um grafo *não* pode ser pintado com três cores: mesmo que
um mago poderosíssimo soubesse que o grafo não pode ser pintado
e desejasse convercer você deste fato ele e você teriam uma tarefa difícil
pela frente.

[]s, N.
=
Instruções para entrar na lista, sair da lista e usar a lista em
http://www.mat.puc-rio.br/~nicolau/olimp/obm-l.html
O administrador desta lista é <[EMAIL PROTECTED]>
=



Re: [obm-l] P e NP

2002-11-11 Por tôpico Nicolau C. Saldanha
On Sun, Nov 10, 2002 at 05:51:19PM -0300, Carlos Maçaranduba wrote:
> Deixa eu ver se entendi bem.Os problemas P são
> resolvidos em tempo aceitavel(porque é da ordem de um
> polinomio)e fornece a resposta procurada com exatidao
> , por isso são deterministicos.Os NP são de ordem
> exponencial e os computadores atuais levariam muito
> tempo para achar a resposta e o que se faz é o uso de
> técnicas probabilisticas(portanto nao deterministicas)
> em tempo polinomial para se achar a resposta.Um
> problema  x NP completo ,é o representante de uma
> classe  de problemas que pódem ser reduzidos a x e
> portanto seriam NP.
> Agora uma coisa que nao ficou clara é por que se
> define NP como uma verificação de resposta e não como
> uma busca de resposta.È porque como a solução é
> probabilistica , dado que eu a achei, devo verificar a
> resposta???Se assim for,toda verificação de resposta é
> em tempo polinomial???Como seria??

Acabo de comentar em outro e-mail. Problemas NP são exatamente
aqueles cuja resposta pode ser verificada em tempo polinomial.
Nem todo problema difícil é assim, claro. Estamos sempre
falando de responder a pergunta com certeza, não de métodos
probabilísticos. []s, N.
=
Instruções para entrar na lista, sair da lista e usar a lista em
http://www.mat.puc-rio.br/~nicolau/olimp/obm-l.html
O administrador desta lista é <[EMAIL PROTECTED]>
=



Re: [obm-l] Livros

2002-11-11 Por tôpico Nicolau C. Saldanha
On Thu, Jan 01, 1998 at 06:04:47AM -0200, Fernando wrote:
> Amigos Virtuais,
> Como poderia adquirir esses livros?
> 
> a.. Olimpíadas Brasileiras de Matemática, 1a. a 8a. :Problemas e Soluções
> Compilado por Élio Mega e Renate Watanabe.
> Sociedade Brasileira de Matemática - SBM.
> 
> 
> a.. Olimpíadas Brasileiras de Matemática, 9a. a 15a.
> Luiz Amancio Machado de Sousa Jr.
> Edições UFC. Fortaleza - CE.
> 
> 
> a.. Olimpíada de Matemática do Estado do Rio de Janeiro (Problemas e Soluções)
> Antonio Luiz Santos, Eduardo Wagner, Raul F. Agostinho
> Sociedade Brasileira de Matemática - SBM.
> 
> 
> a.. Olimpíadas de Matemática 97 - Provas Compiladas e Resolvidas
> Antonio Caminha, Onofre Campos, Paulo Bonfim Gomes Rodrigues
> Editora 7 de Setembro - Fortaleza - CE 
> a.. Atenciosamente, 
> a.. Fernando
> 

Para os livros publicados pela SBM, tente www.sbm.org.br
ou escreva para a secretária da SBM encarregada de venda de livros.

[]s, N.
=
Instruções para entrar na lista, sair da lista e usar a lista em
http://www.mat.puc-rio.br/~nicolau/olimp/obm-l.html
O administrador desta lista é <[EMAIL PROTECTED]>
=



Re: [obm-l] P e NP

2002-11-11 Por tôpico Nicolau C. Saldanha
On Mon, Nov 11, 2002 at 04:28:01PM -0200, Nicolau C. Saldanha wrote:
> On Sun, Nov 10, 2002 at 05:51:19PM -0300, Carlos Maçaranduba wrote:
> > Deixa eu ver se entendi bem.Os problemas P são
> > resolvidos em tempo aceitavel(porque é da ordem de um
> > polinomio)e fornece a resposta procurada com exatidao
> > , por isso são deterministicos.Os NP são de ordem
> > exponencial e os computadores atuais levariam muito
> > tempo para achar a resposta e o que se faz é o uso de
> > técnicas probabilisticas(portanto nao deterministicas)
> > em tempo polinomial para se achar a resposta.Um
> > problema  x NP completo ,é o representante de uma
> > classe  de problemas que pódem ser reduzidos a x e
> > portanto seriam NP.
> > Agora uma coisa que nao ficou clara é por que se
> > define NP como uma verificação de resposta e não como
> > uma busca de resposta.È porque como a solução é
> > probabilistica , dado que eu a achei, devo verificar a
> > resposta???Se assim for,toda verificação de resposta é
> > em tempo polinomial???Como seria??
> 
> Acabo de comentar em outro e-mail. Problemas NP são exatamente
> aqueles cuja resposta pode ser verificada em tempo polinomial.
> Nem todo problema difícil é assim, claro. Estamos sempre
> falando de responder a pergunta com certeza, não de métodos
> probabilísticos. []s, N.

Falando em computadores atuais, computadores quânticos mudariam
muito o nosso poder de resolver problemas. Existe toda uma classe
QP de problemas que são quanticamente polinomiais, i.e., demoram
tempo polinomial em um computador quantico. Conjectura-se que
P esteja estritamente contido em QP que por sua vez estaria
estritamente contido em NP. Um exemplo de problema em QP mas
provavelmente não em P é o de fatorar inteiros. Por outro lado
ninguém sabe provar sequer que P e NP são diferentes então é
possível (mas improvável) que estas três classes afinal de contas
sejam iguais.

Há uma matéria boa sobre computação quântica na última Scientific American.
Vale a pena.

[]s, N.
=========
Instruções para entrar na lista, sair da lista e usar a lista em
http://www.mat.puc-rio.br/~nicolau/olimp/obm-l.html
O administrador desta lista é <[EMAIL PROTECTED]>
=



Re: [obm-l] Teoremas

2002-11-12 Por tôpico Nicolau C. Saldanha
On Tue, Nov 12, 2002 at 12:29:43AM -0200, André Linhares wrote:
> O último teorema de Fermat diz que não existem soluções inteiras para a 
> equação x^z= w^z +y^z com z > 2. Ah! E o endereço do site eh 
> www.teorema.mat.br .

Você se esqueceu de dizer que x, w, e y devem ser não nulos.
Existem sempre as soluções triviais como

1^3 + 0^3 = 1^3

[]s, N.
=
Instruções para entrar na lista, sair da lista e usar a lista em
http://www.mat.puc-rio.br/~nicolau/olimp/obm-l.html
O administrador desta lista é <[EMAIL PROTECTED]>
=



[obm-l] Re: [obm-l] Equação algébrica.

2002-11-12 Por tôpico Nicolau C. Saldanha
On Mon, Nov 11, 2002 at 06:46:52PM -0200, cfgauss77 wrote:
>  Alguém poderia me ajudar a demonstrar se a afirmativa 
> abaixo é verdadeira ou falsa.
>   "Se P(x) e Q(x) são dois polinômios com coeficientes 
> reais e graus iguais a m e n, respectivamente, e M é o 
> maior entre os números m e n, então a equação P(x)=Q(x) 
> tem, no máximo M raízes inteiras e positivas.
>Valeu!!!

(P-Q)(x) é um polinômio de grau no máximo M e tem portanto
no máximo M raízes complexas e em particular no máximo M raízes
inteiras e positivas. []s, N.
=
Instruções para entrar na lista, sair da lista e usar a lista em
http://www.mat.puc-rio.br/~nicolau/olimp/obm-l.html
O administrador desta lista é <[EMAIL PROTECTED]>
=



Re: [obm-l] Teoremas

2002-11-12 Por tôpico Nicolau C. Saldanha
On Mon, Nov 11, 2002 at 10:51:40PM -0200, Wagner wrote:
> Oi pessoal ! 
> 
> Queria saber qual é o último teorema de Fermat e também se existe alguma
> demonstração do teorema de Pitágoras que não use esse mesmo teorema (como
> a dedução da lei dos cossenos por exemplo.

Outras pessoas apontaram para demonstrações usando áreas, como no attachment.
Calcule a área do quadrado grande diretamente para obter (a+b)^2.
Calcule novamente somando as peças para obter c^2 + 2 ab.
Iguale:

a^2 + 2ab + b^2 = c^2 + 2ab
a^2 + b^2 = c^2

Mas as demonstrações mais elementares são por semelhanças.
Na segunda figura é fácil verificar os tamanhos dos segmentos
indicados vendo que os três triangulos retângulos são semelhantes.
A hipotenusa to trângulo grande mede

c = a^2/c + b^2/c

donde

a^2 + b^2 = c^2

[]s, N.



<><>

Re: [obm-l] subconjuntos

2002-11-12 Por tôpico Nicolau C. Saldanha
On Tue, Nov 12, 2002 at 07:03:32AM -0300, Marcos Aurelio Almeida da Silva wrote:
> 
> mas aí você tá contado o par {},{}, que não entra na contagem pois não é
> um par de conjuntos disjuntos...
> 
> > A resposta é a metade de (3^n +1).

Dois conjuntos A e B são disjuntos se A interseção B for igual a vazio.
Mas vazio interseção vazio é igual a vazio. Assim o par {},{} *deve*
ser contado sim. 

Eu gostaria de aproveitar a situação para reforçar o que o Morgado
falou: não deixem de ler as mensagens anteriores. Várias soluções
erradas foram publicadas depois da solução certa do Morgado.
Claro que todo mundo tem o direito de não entender uma solução
ou mesmo de se enganar e achar que o que está certo está errado
mas ignorar completamente uma solução correta é muito desestimulante
para quem, como o Morgado, se deu ao trabalho de responder.
Talvez valha a pena chamar a atenção para o fato de o Morgado
ser um professor respeitado que responde a um monte de perguntas
desta lista sem ter nenhuma obrigação de fazer isso.

[]s, N.
=
Instruções para entrar na lista, sair da lista e usar a lista em
http://www.mat.puc-rio.br/~nicolau/olimp/obm-l.html
O administrador desta lista é <[EMAIL PROTECTED]>
=



Re: [obm-l] QI e outros(Henri Poincare)

2002-11-12 Por tôpico Nicolau C. Saldanha
On Tue, Nov 12, 2002 at 06:12:06PM +, Henrique Lima Santana wrote:
> ô colega,isso q vc colocou ae NÃO sao perguntas de um teste de QI!!!testes 
> de QI sequer teem enunciado...vc provavelmente estah confundindo teste de QI 
> com testes de cultura geral...o problema eh q testes de QI medem APENAS a 
> inteligencia logico-matematica ,nao medindo outras formas de inteligencia 
> como a emocional por exemplo, POR ISSO SAO CRITICADOS PELOS PSICOLOGOS. 
> procure o livro sobre inteligencias multiplas do Howard Gardner...e procure 
> sobre o Raven II Progressive Matrices tbem, provavelmente o mais confiavel 
> no mundo hj. aceito por TODOS os psicologos no mundo.

Com todo o respeito, acho que a coisa está ficando um pouco off-topic, não?

[]s, N.

=
Instruções para entrar na lista, sair da lista e usar a lista em
http://www.mat.puc-rio.br/~nicolau/olimp/obm-l.html
O administrador desta lista é <[EMAIL PROTECTED]>
=



Re: [obm-l] enigma 14-15 de sam loyd

2002-11-13 Por tôpico Nicolau C. Saldanha
On Tue, Nov 12, 2002 at 07:22:39PM +, Henrique Lima Santana wrote:
> 
> 
> ae, Nicolau, qnd vc deu aula sobre invariantes combinatorios no Teorema II 
> em Fortaleza, vc apresentou o enigma 14-15 do sam loyd, nao foi? mas vc nao 
> solucionou...eu achei uma solução meio forçada usando um conceito de 
> parametro de desordem Dp, q soh poderia ser par (em qq posição derivada da 
> resposta), mas eh impar na configuração inicial...existe uma outra solução 
> além desta?
> valeu
> Henrique

Oi Henrique, você está escrevendo na lista. Acho melhor primeiro você
enunciar o problema. Eu mesmo não tenho certeza do que é, talvez seja
aquele jogo com um tabuleiro quadrado 4x4 e 15 quadradinhos numerados
de 1 a 15. A configuração inicial é

 1  2  3  4
 5  6  7  8
 9 10 11 12
13 14 15 --

onde -- representa o único espaço livre. As jogadas válidas consistem
em empurrar um quadradinho viziho para o espaço vago. Assim as posições
válidas a partir da inicial são

 1  2  3  41  2  3  4 
 5  6  7  8   e5  6  7  8
 9 10 11 129 10 11 --
13 14 -- 15   13 14 15 12

e depois disso

 1  2  3  41  2  3  41  2  3  4  1  2  3  4
 5  6  7  8   e5  6  7  8e   5  6  7 -- e5  6  7  8
 9 10 11 129 10 -- 129 10 11  8  9 10 -- 11
13 -- 14 15   13 14 11 15   13 14 15 12 13 14 15 12

A pergunta talvez seja se é possível chegar em 
  
 1  2  3  4
 5  6  7  8
 9 10 11 12
13 15 14 --
  
Se for isso eu resolvo em outra mensagem. Se não for mande a pergunta.
[]s, N.

=
Instruções para entrar na lista, sair da lista e usar a lista em
http://www.mat.puc-rio.br/~nicolau/olimp/obm-l.html
O administrador desta lista é <[EMAIL PROTECTED]>
=



[obm-l] Re: [obm-l] equação

2002-11-14 Por tôpico Nicolau C. Saldanha
On Wed, Nov 13, 2002 at 01:13:16PM -0200, Marcelo Leitner wrote:
> On Wed, Nov 13, 2002 at 10:14:09AM -0500, [EMAIL PROTECTED] wrote:
> > Determine as raízes de z^2+2iz+2-4i=0 sendo i a unidade imaginária. No 
> > gabarito dá
> > 1+i e -1-3i como soluções  e verifica-se que é verdade...mas no braço dá 
> > respostas diferentes ...onde estou errando??
> > Um abraço e um antecipado agradecimento a quem puder elucidar minha 
> > duvida.
> > Korshinói,
> ---end quoted text---
> 
> Ola'!
> Fazendo z=a+bi na equacao aih de cima, obtive o sistema:
> (I)  a^2-b^2-2b+2=0
> (II) 2ab+2a-4=0
> aih isolando a em (II), tem-se: (III) a=2/(b+1)
> Substituindo (III) em (I), tem-se uma parada grande, que
> fatorada sera: (b-1)(b+3)(b+1-i)(b+1+i) = 0
> Como b nao deve ser imaginario, pegamos apenas as 2.
> primeiras raizes, 1 e -3.
> Substituindo elas em (III), chega-se as respostas dadas,
> a=1 p/ b=1 e a=-1 p/ b=-3
> Aih montando-se o z novamente, tem-se:
> z=a+bi
> z_1=1+i e z_2=-1-3i
> 
> Vale apena relembrar que ao fazer (a+bi)^2, o b^2 fica
> negativo, devido ao i^2. (relembro aqui agora pq bobiei
> e fiz o exercicio na primeira vez com b^2 positivo hehehe)

Parece certo, mas não é necessário introduzir a e b no problema.
Você pode simplesmente usar a fórmula que você bem conhece
para resolver equações do segundo grau:

z^2 + (2i) z + (2 - 4i) = 0

z = -2i +- sqrt((2i)^2 - 4(2 - 4i))/2

z = -i +- sqrt(-1 - 2 + 4i)

z = -i +- sqrt(-3+4i)

(Talvez a dificuldade seja tirar a raiz quadrada?  Dá 1+2i.)

z = 1+i, z = -1-3i

Ou, como você tem as raízes, basta verificar que a soma e o produto
são a menos de sinais os coeficientes da equação:

(1+i) + (-1-3i) = -(2i)
(1+i)(-1-3i) = (2 - 4i)

[]s, N.


=====
Instruções para entrar na lista, sair da lista e usar a lista em
http://www.mat.puc-rio.br/~nicolau/olimp/obm-l.html
O administrador desta lista é <[EMAIL PROTECTED]>
=



[obm-l] Re: [obm-l] Re: [obm-l] equação

2002-11-14 Por tôpico Nicolau C. Saldanha
On Thu, Nov 14, 2002 at 10:34:27AM -0200, Marcelo Leitner wrote:
> Exatamente, eu nao tinha enxergado que (-3+4i) = (1+2i)^2, aih
> optei pelo metodo mais "generico"..
> Tem algum jeito de identificar essa fatoracao jah de primeira
> vista ou eh soh conhecendo elas mesmo?

Uma opção é usar coordenadas polares:
sqrt(r (cos t + i sen t)) = sqrt(r) (cos (t/2) + i sen (t/2))

Outra opção é fazer o tipo de coisa que você fez: 
vamos resolver

a+bi = (c+di)^2 = (c^2 - d^2) + 2cd i

donde

c^2 - d^2 = a
2cd = b

temos

d = b/(2c)

e

c^2 - b^2/(4 c^2) = a

Assim c é uma das raízes reais da equação biquadrada

4 c^4 - 4 a c^2 - b^2 = 0

Ou seja 

c = +- sqrt( (  a + sqrt(a^2 + b^2)) / 2 )
d = +- sqrt( (- a + sqrt(a^2 + b^2)) / 2 )

[]s, N.
=
Instruções para entrar na lista, sair da lista e usar a lista em
http://www.mat.puc-rio.br/~nicolau/olimp/obm-l.html
O administrador desta lista é <[EMAIL PROTECTED]>
=



[obm-l] Re: [obm-l] Re: [obm-l] equação

2002-11-14 Por tôpico Nicolau C. Saldanha
On Thu, Nov 14, 2002 at 11:51:09AM -0200, Augusto Cesar de Oliveira Morgado wrote:
> > ATÉ VOCÊ, NICOLAU? Estou me sentindo um homem invisível!
> Morgado

Foi mal, eu deixei a mensagem pela metade ontem de noite e quando
voltei continuei sem ver a sua. Pessoalmente você não é nada invisível! ;-)

[]s, N.

=
Instruções para entrar na lista, sair da lista e usar a lista em
http://www.mat.puc-rio.br/~nicolau/olimp/obm-l.html
O administrador desta lista é <[EMAIL PROTECTED]>
=



Re: [obm-l] enigma 14-15 de sam loyd

2002-11-14 Por tôpico Nicolau C. Saldanha
On Wed, Nov 13, 2002 at 01:00:58PM +, Henrique Lima Santana wrote:
> soh, na verdade, o original era com o nº 14 no lugar do 15 e o 15 no lugar 
> do 14,i.e., começa onde vc colocou q termina e termina onde vc
> disse q começa...de fato, o problema q vc passou no II Teorema foi esse aí 
> de baixo(acabei de ver minhas anotações aqui)...de qquer forma, do jeito q 
> vc colocou, ele a principio tem Dp par e qq mudança q vc faça vai manter o 
> Dp par(eh invariante!) ; no entanto a configuração q vc quer tem Dp ímpar, 
> por isso eh absurdo!
> blz entao, manda tua resposta,falou?
> valeu
> Henrique

Não entendi bem a sua solução. O que é Dp? Talvez um determinante?
Talvez a sua solução seja parecida com a minha, em todo caso...

A minha solução envolve o conceito de permutação par e ímpar
(em outra mensagem eu posso explicar o que é uma permutação par ou ímpar).

Pinte os quadrados de fundo do tabuleiro de preto e branco assim:

 B P B P
 P B P B
 B P B P
 P B P B

A cada movimento o buraco muda de cor (inicialmente ele é branco,
depois preto, depois branco,...). Se pensarmos no buraco como um 16
e lermos o quadro como uma permutação de {1,2,...,16} a cada movimento
esta permutação troca de paridade (pois trocamos o 16 de lugar com alguém).
Inicialmente ela é par (a identidade!), depois ímpar, depois par,...
Assim a permutação é par sse o buraco é branco. Na configuração onde
estamos tentando chegar a permutação é ímpar e o buraco é branco
donde esta configuração nunca pode ser alcançada.

[]s, N.
=
Instruções para entrar na lista, sair da lista e usar a lista em
http://www.mat.puc-rio.br/~nicolau/olimp/obm-l.html
O administrador desta lista é <[EMAIL PROTECTED]>
=



[obm-l] Re: [obm-l] Re: [obm-l] Re: [obm-l] equação

2002-11-14 Por tôpico Nicolau C. Saldanha
On Thu, Nov 14, 2002 at 04:43:38PM -0200, Nicolau C. Saldanha wrote:
> On Thu, Nov 14, 2002 at 10:34:27AM -0200, Marcelo Leitner wrote:
> > Exatamente, eu nao tinha enxergado que (-3+4i) = (1+2i)^2, aih
> > optei pelo metodo mais "generico"..
> > Tem algum jeito de identificar essa fatoracao jah de primeira
> > vista ou eh soh conhecendo elas mesmo?
> 
> Uma opção é usar coordenadas polares:
> sqrt(r (cos t + i sen t)) = sqrt(r) (cos (t/2) + i sen (t/2))
> 
> Outra opção é fazer o tipo de coisa que você fez: 
> vamos resolver
> 
> a+bi = (c+di)^2 = (c^2 - d^2) + 2cd i
> 
> donde
> 
> c^2 - d^2 = a
> 2cd = b
> 
> temos
> 
> d = b/(2c)
> 
> e
> 
> c^2 - b^2/(4 c^2) = a
> 
> Assim c é uma das raízes reais da equação biquadrada
> 
> 4 c^4 - 4 a c^2 - b^2 = 0
> 
> Ou seja 
> 
> c = +- sqrt( (  a + sqrt(a^2 + b^2)) / 2 )
> d = +- sqrt( (- a + sqrt(a^2 + b^2)) / 2 )

Andei repetindo o Morgado de novo! Oops...

Mas desta vez ele só deu a fórmula, e eu mostrei como chegar nela. Ha!
Talvez isso tenha algo a ver com o fato de que eu não sabia a fórmula,
eu a obtive enquanto escrevia o e-mail.

De qualquer forma, vamos todos ler o que o Morgado escreve.
Vale a pena.

[]s, N.


=========
Instruções para entrar na lista, sair da lista e usar a lista em
http://www.mat.puc-rio.br/~nicolau/olimp/obm-l.html
O administrador desta lista é <[EMAIL PROTECTED]>
=



Re: 14/11 [obm-l] QI e outros

2002-11-18 Por tôpico Nicolau C. Saldanha
On Thu, Nov 14, 2002 at 06:03:18PM -0300, JOÃO CARLOS PAREDE wrote:
> 
> O teste de QI é baseado a partir da teoria Behaviorista, ...

Este assunto é off topic. Se há gente aqui interessada nele
vocês podem se comunicar de mil outras formas mas por favor
não descaracterizem esta lista.

Obrigado, []s, N.
=
Instruções para entrar na lista, sair da lista e usar a lista em
http://www.mat.puc-rio.br/~nicolau/olimp/obm-l.html
O administrador desta lista é <[EMAIL PROTECTED]>
=



Re: [obm-l] virus?

2002-11-18 Por tôpico Nicolau C. Saldanha
On Fri, Nov 15, 2002 at 12:09:13AM -0200, Margarida Lanna wrote:
> Alguém da área de informatica pode me ajudar?
> 
> Os ícones da minha área de trabalho estão se movendo loucamente de acordo com o 
>movimento do mouse. Seria algum vírus?

Totalmente off topic, por favor... []s, N.


=
Instruções para entrar na lista, sair da lista e usar a lista em
http://www.mat.puc-rio.br/~nicolau/olimp/obm-l.html
O administrador desta lista é <[EMAIL PROTECTED]>
=



Re: 14/11[obm-l] Teoremas

2002-11-18 Por tôpico Nicolau C. Saldanha
On Thu, Nov 14, 2002 at 06:06:28PM -0300, JOÃO CARLOS PAREDE wrote:
> 
> ÚLTIMO TEOREMA DE FERMAT
> Não existem soluções inteiras para a equação
> x^n + y^n = z^n
> para n > 2.

Faltou dizer que x, y e z são todos não nulos
(caso contrário é obviamente falso).
Este mesmo erro já foi cometido e sua correção
foi feita nesta lista há umas duas semanas.

[]s, N.
=
Instruções para entrar na lista, sair da lista e usar a lista em
http://www.mat.puc-rio.br/~nicolau/olimp/obm-l.html
O administrador desta lista é <[EMAIL PROTECTED]>
=



[obm-l] Re: your mail

2002-11-18 Por tôpico Nicolau C. Saldanha
On Fri, Nov 15, 2002 at 07:35:19PM +, Roberto Gomes wrote:
> Qual programa que ler arquivos *.ps?

Esta pergunta é off topic e aparece muito nesta lista.
Verifique os arquivos para o mês de outubro ou tente procurar
por 'PostScript' em www.google.com.

[]s, N.
=
Instruções para entrar na lista, sair da lista e usar a lista em
http://www.mat.puc-rio.br/~nicolau/olimp/obm-l.html
O administrador desta lista é <[EMAIL PROTECTED]>
=



[obm-l] Re: [obm-l] É sempre possível?

2002-11-18 Por tôpico Nicolau C. Saldanha
On Fri, Nov 15, 2002 at 05:55:54PM -0300, cgmat wrote:
> Uma questão proprosta enum vestibular dizia que:
> 
>  Seja n um número inteiro e positivo. Se n é par divida-o por 2; se n
>  é ímpar, multiplique-o por 3 e adicione 1 ao resultado. Esse
>  procedimento deve ser repetido até que o resultado  final seja 1.
>  Assim , por exemplo se n=12, tem-se
> 
> 12  -  6  -  3  -  10  -  5  -  16  -  8  -  4  -  2  -  1
> 
> ou seja, foram necessárias 9 passagens até obter-se o resultado 1. Se
> n=11 quantas passagens seriam necessárias para obter o resiultado
> final 1?  Bem é trivial descobrir que são necessárias 14 passagens. A
> minha dúvida é dado qualquer inteiro positivo n é  sempre possível
> através das operações acima definidas chegar ao resultado 1? E neste
> caso , se possível, como determinar a quandidade de " passagens"
> necessárias para isso sem ter que descrevê-las?

Este é um problema em aberto bem conhecido.
Não apenas ninguém sabe responder a sua dúvida como ninguém
tem nenhuma idéia plausível de como se poderia tentar responder.

[]s, N.
=
Instruções para entrar na lista, sair da lista e usar a lista em
http://www.mat.puc-rio.br/~nicolau/olimp/obm-l.html
O administrador desta lista é <[EMAIL PROTECTED]>
=



[obm-l] Re: [obm-l] nºs de bernoulli

2002-11-18 Por tôpico Nicolau C. Saldanha
On Fri, Nov 15, 2002 at 04:37:09PM +, Henrique Lima Santana wrote:
> 
> ae, alguem pode me definir os nºs de Bernoulli ?


Uma definição simples é a seguinte

t/(e^t - 1) = sum_k B_k/k! t^k

Os primeiros valores são B_0 = 1, B_1 = -1/2, B_2 = 1/6,...
Temos B_(2k+1) = 0 para k > 1 e (-1)^(k+1) B_(2k) > 0 para k >= 1.

Eles aparecem em um monte de séries como em

tan t = sum_{k >= 1} 2^(2k)(2^(2k) - 1)B_(2k)/(2k)! t^(2k-1)

Os polinômios de Bernoulli podem ser definidos por

te^(xt)/(e^t - 1) = sum_k B_k(x)/k! t^k

e satisfazem

B_n(0) = B_n

B_n(x+1) - B_n(x) = n x^(n-1)

(B_(n+1)(k+1) - B_(n+1)(0))/(n+1) = 1^n + 2^n + ... + k^n

> outra coisa, como se
> prova q Cn=C2n,n/(n+1) onde Cn=n-esimo nº de Catalan ? (eh isso
> mesmo?)

Para mim isto é a definição de número de Catalan.
Há muuuitas interpretações combinatórias para números de Catalan.

> alguem ae jah estudou soluçoes em inteiros pra equaçao x^l+y^l=cz^l , ou 
> melhor, x^l+y^l=2z^l pra l primo >7 .

A única coisa que me ocorre é que x=y=z é solução. :-)
>   falou
>   Henrique (ah,acabei de ver um problema legal,alguem conhece? !p
> eh incongruente a 0 mod p pra todo primo p impar)

Não entendi nada. O que significa !p ?

[]s, N.
=
Instruções para entrar na lista, sair da lista e usar a lista em
http://www.mat.puc-rio.br/~nicolau/olimp/obm-l.html
O administrador desta lista é <[EMAIL PROTECTED]>
=



Re: [obm-l] z^z

2002-11-18 Por tôpico Nicolau C. Saldanha
On Mon, Nov 18, 2002 at 10:30:40AM -0200, Augusto César Morgado wrote:
>   z1^z2 =  exp (z2 * ln z1)
> 
> Jose Francisco Guimaraes Costa wrote:
> 
> > Sejam z1 e z2 dois números complexos.
> >
> >  
> >
> > A operação z1^z2 é definida? Se for, qual sua definição?

A definição do Morgado é ótima mas é preciso chamar a atenção para
o fato de ln z1 não estar tão bem definido assim. A função ln não
pode ser definida assim

ln : C - {0} -> C

precisamos fazer um corte, como por exemplo

ln : C - {z in R, z <= 0} -> C

e escolhas diferentes do corte produzem valores diferentes para ln z1.

[]s, N.
=
Instruções para entrar na lista, sair da lista e usar a lista em
http://www.mat.puc-rio.br/~nicolau/olimp/obm-l.html
O administrador desta lista é <[EMAIL PROTECTED]>
=



Re: [obm-l] (nenhum assunto)

2002-11-18 Por tôpico Nicolau C. Saldanha
On Sun, Nov 17, 2002 at 09:45:42PM -0500, [EMAIL PROTECTED] wrote:
> Não estou mais recebendo mensagens da lista...algum problema temporário?? 
> Será que devo me inscrever de novo??
> Se alguem pudesse me explicar eu agradeceria
>korshinoi

Nosso servidor teve problemas durante o fim de semana prolongado de 15 a 17.
Como vocês podem ver, estamos no ar de novo.

[]s, N.
=
Instruções para entrar na lista, sair da lista e usar a lista em
http://www.mat.puc-rio.br/~nicolau/olimp/obm-l.html
O administrador desta lista é <[EMAIL PROTECTED]>
=



Re: [obm-l] Toro x Esfera

2002-11-19 Por tôpico Nicolau C. Saldanha
On Tue, Nov 19, 2002 at 11:32:36AM -0300, bruno lima wrote:
>Existe aplicação contínua da esfera (S^2) no
> Toro(S^1 X S^1) ?  E o contrario ??
>  me parece que nao mas nao sei como provar

Claro que sim, você pode tomar a aplicação constante. Ao contrário também.

Eu suspeito que você queria perguntar outra coisa.
Talvez você estivesse pensando se existe uma função contínua
e bijetora da esfera para o toro? Neste caso a resposta
é não mas por motivos bem menos elementares:
a esfera é simplesmente conexa e o toro não.

[]s, N.
=
Instruções para entrar na lista, sair da lista e usar a lista em
http://www.mat.puc-rio.br/~nicolau/olimp/obm-l.html
O administrador desta lista é <[EMAIL PROTECTED]>
=



Re: [obm-l] ???

2002-11-19 Por tôpico Nicolau C. Saldanha
On Tue, Nov 19, 2002 at 03:04:46PM -0200, Eder wrote:
> Olá,
> 
> Eu gostaria de passar um probleminha que não vi ainda como resolver:
> 
> 
> Dois colecionadores de selos têm,juntos,500 selos.Cada colecionador comprou
> um álbum para colocar seus selos.Os dois álbuns eram idênticos,tendo o mesmo
> número de páginas.
> 
> Se o primeiro colecionador colocar exatamente 21 selos em cada página,ele vai
> conseguir colocar todos os seus selos e usar todas as páginas do álbum.
> 
> Se o segundo colecionador colocar 20 de seus selos em cada página do
> álbum,sobrarão alguns selos.Caso ele coloque 23 selos em cada página,sobra
> pelo menos uma,totalmente vazia,podendo haver ainda uma outra página com
> menos de 20 selos.
> 
> Quantas páginas há no álbum???

Seja n o número de páginas, a e b os números de selos dos dois colecionadores
(na ordem do problema).

Os dados são:

(0)a + b = 500

(1)a = 21n
(2)b > 20n
(3)b < 23(n-2) + 20

somando (1) com (2) e (1) com (3) e usando (0)
para substituir a+b por 500 temos

(4)  500 > 41n
(5)  500 < 44n - 26

ou

(6)  526/44 < n < 500/41

Fazendo as contas é fácil ver que n = 12, a = 252, b = 248.

[]s, N.



=
Instruções para entrar na lista, sair da lista e usar a lista em
http://www.mat.puc-rio.br/~nicolau/olimp/obm-l.html
O administrador desta lista é <[EMAIL PROTECTED]>
=



[obm-l] Re: [obm-l] ---> Dúvida

2002-11-21 Por tôpico Nicolau C. Saldanha
On Tue, Nov 19, 2002 at 07:09:55PM -0200, cfgauss77 wrote:
>   Gostaria de saber se posso enviar questões à Lista por 
> meio de arquivos .DOC anexados.

Não. Os motivos para isso estão explicitados em
http://www.mat.puc-rio.br/~nicolau/olimp/obm-l.html

Se você tentar enviar o seu e-mail será barrado.
Se eu estiver de muito bom humor eu devolvo
o e-mail com uma explicação mas normalmente
eu simplesmente jogo fora.

[]s, N.
=
Instruções para entrar na lista, sair da lista e usar a lista em
http://www.mat.puc-rio.br/~nicolau/olimp/obm-l.html
O administrador desta lista é <[EMAIL PROTECTED]>
=



[obm-l] Re: [obm-l] tan t [era nºs de bernoulli]

2002-11-21 Por tôpico Nicolau C. Saldanha
On Tue, Nov 19, 2002 at 07:31:46PM -0200, Luis Lopes wrote:
> Sauda,c~oes,
> 
> Eles aparecem em um monte de séries como em
> >
> > tan t = sum_{k >= 1} 2^(2k)(2^(2k) - 1)B_(2k)/(2k)! t^(2k-1)
> >
> Justamente, outro dia estava querendo saber
> como obter esta série. Referências?

Para não errar eu olhei no Encyclopaedic Dictionary of MAthematics
mas você deve encontrar isso em bons (realmente bons) livros de cálculo.
Ou você pode tentar deduzir sozinho, não é tão difícil assim,
especialmente sabendo a resposta.

[]s, N.
=
Instruções para entrar na lista, sair da lista e usar a lista em
http://www.mat.puc-rio.br/~nicolau/olimp/obm-l.html
O administrador desta lista é <[EMAIL PROTECTED]>
=



Re: [obm-l] z^z - mais perguntas

2002-11-21 Por tôpico Nicolau C. Saldanha
On Wed, Nov 20, 2002 at 09:36:19PM -0400, Jose Francisco Guimaraes Costa wrote:
> (1) Usando a mesma linguagem segundo a qual a expressão 
> 
> A = sqrt(B)
> 
> é lida como "A é igual à raiz quadrada de B", como ler a expressão
> 
> ln : C - {z in R, z <= 0} -> C   ?

A função ln tem como domínio o seguinte conjunto X de números complexos.
Todos os números complexos não reais pertencem a X;
os números reais estritamente positivos também pertencem a X;
o número 0 e os reais negativos não pertencem a X.
 
> (2) N diz "precisamos fazer um corte, como por exemplo ... ". Por que
> precisamos fazer um corte (ou por que "A função ln não pode ser definida
> assim: ln : C - {0} -> C") ?

Não existe uma função contínua f: C - {0} -> C satisfazendo
exp(f(z)) = z para todo z in C - {0}.

O problema é com a parte imaginária de ln z que é o argumento de z
pois se z = r e^(it) queremos definir ln(z) = ln(r) + it.
Não podemos definir continuamente o argumento pois quando damos
uma volta completa o argumento deve aumentar de 2 Pi e ficar constante
ao mesmo tempo o que é um absurdo.
 
> (3) A afirmação "precisamos fazer um corte, como por exemplo ... e escolhas
> diferentes do corte produzem valores diferentes para ln z" me deixa com a
> idéia de que eu posso escolher o corte que me convier, o que faz com que a
> função "ln z" não tenha uma definição única. É isso mesmo?

Para qualquer conjunto aberto e simplesmente conexo X contido em C
com 0 não pertencente a X e 1 pertence a X existe uma única função
contínua f: X -> C satisfazendo f(1) = 0 e exp(f(z)) = z para todo
z in X.

De novo a questão é definir continuamente o argumento.

> (4) Faz sentido dizer que um número complexo é positivo ou negativo? Se
> fizer, quando ele é positivo e quando é negativo?

Não existe nenhuma definição útil ou usual de número complexo positivo.
Para mim quando se escreve 'z > 0' o que se está dizendo implicitamente
é 'z é real e z > 0'.

> (5) Por favor sugiram livros onde eu possa encontrar respostas para este tipo
> de perguntas. Embora eu tenha estudado números complexos e trabalhado com
> eles - sou engenheiro eletrônico - não me lembro de ter sido exposto às
> definições e conceitos acima.

O Morgado já indicou dois livros excelentes,
o Churchill e o Ahlfors (Complex Analysis).
Um livro diferente que talvez interesse é o Henrici,
Applied and Computational Complex Analysis (3 vols).

[]s, N.
=========
Instruções para entrar na lista, sair da lista e usar a lista em
http://www.mat.puc-rio.br/~nicolau/olimp/obm-l.html
O administrador desta lista é <[EMAIL PROTECTED]>
=



[obm-l] Re: [obm-l] Dúvida

2002-11-21 Por tôpico Nicolau C. Saldanha
On Thu, Nov 21, 2002 at 11:54:12AM -0300, Johann Peter Gustav Lejeune Dirichlet wrote:
> 
> Pergunta:
> E se for algo essencial,como um desenho de um grafico ou de geometria,
> e valido?

Em nenhum caso você deve enviar arquivos *doc.
Você pode enviar arquivos *png, *jpg ou *gif (bem simples)
se você julgar que uma figura é "essencial".

> Afinal explicar geometria sem desenho e coisa de enunciado de IMO.

Com o perdão do sarcasmo, você já se perguntou o que significa
o nome "obm-l"?

[]s, N.


=
Instruções para entrar na lista, sair da lista e usar a lista em
http://www.mat.puc-rio.br/~nicolau/olimp/obm-l.html
O administrador desta lista é <[EMAIL PROTECTED]>
=



[obm-l] Serie de tan t [era nºs de bernoulli]

2002-11-22 Por tôpico Nicolau C. Saldanha
On Thu, Nov 21, 2002 at 06:39:35PM -0200, Luis Lopes wrote:
> > mas você deve encontrar isso em bons (realmente bons) > livros de cálculo.
> É possível, mas nunca vi. Também é verdade que
> nunca os consultei tendo este problema em mente.
> De qualquer jeito...
> 
> > Ou você pode tentar deduzir sozinho, não é tão difícil
> > assim, especialmente sabendo a resposta.
>  é isso que procuro. Em praticamente todos os
> livros de cálculo encontramos as mesmas séries:
> ln(1+x), Arctan x, e^x, cosh x, sinh x, cos x, sin x (ver
> Adams, Robert, Single-Variable Calculus).

Encontrei no livro Concrete Mathematics, de Graham, Knuth, Patashnik,
uma seção sobre números de Bernoulli que deve te interessar.
Vou isolar um esboço da demonstração que você quer. Por definição,

t/(e^t - 1) = sum_k B_k/k! t^k

mas

t/(e^t - 1) + t/2 = (t(2 + e^t - 1))/(2(e^t - 1))
  = t/2 * (e^(t/2) - e^(-t/2))/(e^(t/2) + e^(-t/2))
  = t/2 * coth(t/2)

é ímpar donde

(t/2) * coth(t/2) = sum_k B_(2k)/(2k)! t^(2k)
 
t * coth t = sum_k 2^(2k) B_(2k)/(2k)! t^(2k)

cot t = sum_{k >= 0} (-1)^k 2^(2k) B_(2k)/(2k)! t^(2k-1)

mas

tan t = cot t - 2 cot 2t

  = sum_{k >= 0} (-1)^k 2^(2k) B_(2k)/(2k)! ( t^(2k-1) - 2 (2t)^(2k-1) )

  = sum_{k > 0} (-1)^k 2^(2k) (1 - 2^(2k)) B_(2k)/(2k)! t^(2k-1)

[]s, N.


=
Instruções para entrar na lista, sair da lista e usar a lista em
http://www.mat.puc-rio.br/~nicolau/olimp/obm-l.html
O administrador desta lista é <[EMAIL PROTECTED]>
=



Re: [obm-l] PAs de ordens>1

2002-11-22 Por tôpico Nicolau C. Saldanha
On Fri, Nov 22, 2002 at 02:02:48AM -0300, Alexandre Tessarollo wrote:
> 
>Estou num momento de diarréia mental. Qual é e como deduzir a fórmula de
>somatório de x^2, para x=1,2,..,n? 

O somatório

1^n + 2^n + ... + k^n

onde pensamos em n como um parâmetro fixo mas arbitrário e em k como a variável
é dado por um polinômio de grau (n+1) na variável k:

1^n + 2^n + ... + k^n = a_{n,n+1} k^(n+1) + a_{n,n} k^n + ... + a_{n,0}

Estes coeficientes a_{i,j} são números racionais e você pode sem dificuldade
deduzir várias propriedades deles mas de fato estes são (a menos alterações
mínimas) os polinômios de Bernoulli de que falava há poucos dias atrás;
repetindo...

Os polinômios de Bernoulli podem ser definidos por

te^(xt)/(e^t - 1) = sum_k B_k(x)/k! t^k

e satisfazem

B_n(0) = B_n

B_n(x+1) - B_n(x) = n x^(n-1)

(B_(n+1)(k+1) - B_(n+1)(0))/(n+1) = 1^n + 2^n + ... + k^n

A seção sobre números de Bernoulli do Concrete Mathematics usa exatamente
este problema para motivar o estudo de números de Bernoulli.

[]s, N.
=
Instruções para entrar na lista, sair da lista e usar a lista em
http://www.mat.puc-rio.br/~nicolau/olimp/obm-l.html
O administrador desta lista é <[EMAIL PROTECTED]>
=



[obm-l] Re: [obm-l] Re: [obm-l] Serie de tan t [era nºs de bernoulli]

2002-11-22 Por tôpico Nicolau C. Saldanha
On Fri, Nov 22, 2002 at 03:03:37PM -0200, Luis Lopes wrote:
> Sauda,c~oes,
> 
> Obrigado, Nicolau.
> 
> Vou olhar pela n-ésima vez o livro do Knuth
> e outros.
> 
> Vendo sua demonstração lembrei-me de
> duas folhas que xeroquei do livro
> A Classical Introduction to Modern Number
> Theory by K. Ireland and M. Rosen,
> Springer-Verlag, 1990.
> As folhas reproduzem as páginas 228-231
> do capítulo 15 Bernoulli Numbers.
> Lá vemos a definição e aplicações dos números
> de Ber. A mais elementar é o cálculo das
> somas 1^m + ... + (n-1)^m.

Soma esta que por acaso está sendo discutida agora mesmo nesta lista...

> Preciso agora entender a passagem
> 
> > t * coth t = sum_k 2^(2k) B_(2k)/(2k)! t^(2k)
> >
> > cot t = sum_{k >= 0} (-1)^k 2^(2k) B_(2k)/(2k)! t^(2k-1)

Lembre-se que

coth t = (e^t + e^(-t))/(e^t - e^(-t))

i coth it = cot t = i * (e^(it) + e^(-it))/(e^(it) - e^(-it))

Troque t por it na primeira fórmula...
 
> No momento faço duas correções de teclado (typos):
> 
> > t/(e^t - 1) + t/2 = (t(2 + e^t - 1))/(2(e^t - 1))
> >   = t/2 * (e^(t/2) - e^(-t/2))/(e^(t/2) + e^(-t/2))
> >   = t/2 * coth(t/2)
> 
> A 2a. linha deveria ser
> 
> >   = t/2 * (e^(t/2) + e^(-t/2))/(e^(t/2) - e^(-t/2))
> 
> > é ímpar
> Leia-se é par.

Você tem razão nos dois casos, obrigado pela correção e pela leitura atenta.

[]s, N.
=====
Instruções para entrar na lista, sair da lista e usar a lista em
http://www.mat.puc-rio.br/~nicolau/olimp/obm-l.html
O administrador desta lista é <[EMAIL PROTECTED]>
=



[obm-l] Re: [obm-l] Re: [obm-l] Serie de tan t [era nºs de bernoulli]

2002-11-22 Por tôpico Nicolau C. Saldanha
On Fri, Nov 22, 2002 at 03:03:37PM -0200, Luis Lopes wrote:
> Sauda,c~oes,
> 
> Obrigado, Nicolau.
> 
> Vou olhar pela n-ésima vez o livro do Knuth
> e outros.

Eu tinha dito que você encontraria a série da tangente em
um "bom livro de cálculo" e dei como referência o Concrete Mathematics,
que é um bom livro mas não é um livro de cálculo...

Então para não ficar devendo fui procurar outra referência.
Encontrei a série da tangente no livro "Calculus of one variable",
de Joseph W. Kitchen, Jr., Addison-Wesley, seção 13.7 (Real analytic
functions), página 667. Ele apresenta os números de Bernoulli na
página 665 exatamente para achar as séries de x coth x e x cot x.

[]s, N.
=
Instruções para entrar na lista, sair da lista e usar a lista em
http://www.mat.puc-rio.br/~nicolau/olimp/obm-l.html
O administrador desta lista é <[EMAIL PROTECTED]>
=



Re: [obm-l] IME 96

2002-11-25 Por tôpico Nicolau C. Saldanha
On Sun, Nov 24, 2002 at 08:59:34PM -0300, Eduardo Casagrande Stabel wrote:
> Olá,
> 
> essa questão também caiu na Olimpíada Gaúcha de Matemática. Eu pensei na
> mesma solução da banca. Mas uma das alunas que fez a prova deu uma solução
> mais simples, e que eu achei até mais apropriada ao tamanho do tabuleiro. Ela
> começou escrevendo um 1 no canto superior esquerdo. Para cada quadrado
> seguinte ela preenchia ele com a soma dos números escritos nos quadrados da
> esq. da dir. e da diagonal superior esq. Assim ela foi preenchendo o
> tabuleiro e o número final obtido no inferior direito foi a quantidade de
> maneiras de se chegar até ele.
> 
> Interessante, né?

Também é interessante o fato de que esta tabela apareceu como matriz
na OBM nível U de 2001. Os números que preenchem a tabela são chamados
de números de Delonnay e aparecem em alguns problemas de combinatória.
Na OBM pedia-se para calcular o determinante da matriz (em função de n,
o tamanho da matriz).

[]s, N.

=
Instruções para entrar na lista, sair da lista e usar a lista em
http://www.mat.puc-rio.br/~nicolau/olimp/obm-l.html
O administrador desta lista é <[EMAIL PROTECTED]>
=



<    2   3   4   5   6   7   8   9   10   11   >